CMMKA MedSurg Exam Four

Pataasin ang iyong marka sa homework at exams ngayon gamit ang Quizwiz!

A patient with mild iatrogenic Cushing syndrome is on an alternate-day regimen of corticosteroid therapy. What does the nurse explain to the patient about this regimen? a. It maintains normal adrenal hormone balance. b. It prevents ACTH release from the pituitary gland. c. It minimizes hypothalamic-pituitary-adrenal suppression. d. It provides a more effective therapeutic effect of the drug.

c. Taking corticosteroids on an alternate-day schedule for pharmacologic purposes is less likely to suppress ACTH production from the pituitary and prevent adrenal atrophy. Normal adrenal hormone balance is not maintained during glucocorticoid therapy because excessive exogenous hormone is used.

A patient with hypoparathyroidism resulting from surgical treatment of hyperparathyroidism is preparing for discharge. What should the nurse teach the patient? a. Milk and milk products should be increased in the diet. b. Parenteral replacement of parathyroid hormone will be required for life. c. Calcium supplements with vitamin D can effectively maintain calcium balance. d. Bran and whole-grain foods should be used to prevent GI effects of replacement therapy.

c. The hypocalcemia that results from PTH deficiency is controlled with calcium and vitamin D supplementation and possibly oral phosphate binders. Replacement with PTH is not used because of antibody formation to PTH, the need for parenteral administration, and cost. Milk products, although good sources of calcium, also have high levels of phosphate, which reduce calcium absorption. Whole grains and foods containing oxalic acid also impair calcium absorption.

During assessment of the patient with acromegaly, what should the nurse expect the patient to report? a. Infertility b. Dry, irritated skin m c. Undesirable changes in appearance d. An increase in height of 2 to 3 inches a year

c. The increased production of GH in acromegaly causes an increase in thickness and width of bones and enlargement of soft tissues, resulting in marked changes in facial features, oily and coarse skin, and speech difficulties. Infertility is not a common finding because GH is usually the only pituitary hormone involved in acromegaly. Height is not increased in adults with GH excess because the epiphyses of the bones are closed.

A patient suspected of having acromegaly has an elevated plasma growth hormone (GH) level. In acromegaly, the nurse would also expect the patient's diagnostic results to include a. hyperinsulinemia b. a plasma glucose of < 70 mg/dL c. decreased GH levels with an oral glucose challenge test d. elevated serum somatomedin C (insulin like growth factor-1 (IGF-1))

D- A normal response to growth hormone (GH) secretion is stimulation of the liver to produce somatomedin C, or IGF-1, which stimulates growth of bones and soft tissues. The increased levels of somatomedin C normally inhibit GH, but in acromegaly, the pituitary gland secretes GH despite elevated IGF-1 levels. When both GH and IGF-1 levels are increased, overproduction of GH is confirmed. GH also causes elevation of blood glucose, and normally GH levels fall during an oral glucose challenge but not in acromegaly.

In a patient with central diabetes insipidus, administration of ADH during a water deprivation test will result in a(n) a. decrease in body weight b. increase in urinary output c. decrease in blood pressure d. increase in urine osmolality

D- A patient with diabetes insipidus has a deficiency of ADH with excessive loss of water from the kidney, hypovolemia, hypernatremia, and dilute urine with low specific gravity. When a vasopressin is administered, the symptoms are reversed, with water retention, decreased urinary output that increases urine osmolality, and an increase in BP

When replacement therapy is started for a patient with long-standing hypothyroidism, it is most important for the nurse to monitor the patient for a. insomnia b. weight loss c. nervousness d. dysrhythmias

D- All these manifestations may occur with treatment of hypothyroidism, but as a result of the effects of hypothyroidism on the cardiovascular system, when thyroid replacement therapy is started, myocardial oxygen consumption is increased, and resultant oxygen demand may cause angina, cardiac dysrhythmias, and heart failure

Causes of primary hypothyroidism in adults include a. malignant or benign thyroid nodules b. surgical removal or failure of pituitary gland c. surgical removal or radiation of thyroid gland d. autoimmune induced atrophy of the thryoid gland.

D- Both Grave's disease and Hashimoto's thyroiditis are autoimmune disorders that eventually destroy the thyroid gland, leading to primary hypothyroidism. Thyroid tumors most often result in hyperthyroidism. Secondary hyperthyroidism occurs as a result of pituitary failure, and iatrogenic hypothyroidism results from thyroidectomy or radiation of thyroid gland.

A patient with Grave's disease asks the nurse what caused the disorder. The best response by the nurse is, a. "The cause of Grave's disease is not known, although it is thought to be genetic" b. "It is usually associated with goiter formation from an iodine deficiency over a long period of time" c. "Antibodies develop against thyroid tissue and destroy it, causing a deficiency of thyroid hormones" d. "In genetically susceptible persons, antibodies are formed that cause excessive thyroid hormone secretion"

D- In Grave's disease, antibodies to the TSH receptor are formed, attach to the receptors, and stimulate the thyroid gland to release triiodothyronin (T3), thyroxine (T4), or both, creating hyperthyroidism. The disease is not directly genetic, but individuals appear to have a genetic susceptibility to develop autoimmune antibodies. Goiter formation from insufficient iodine intake is usually associated with hypothyroidism.

When providing discharge instructions to a patient following a subtotal thyroidectomy, the nurse advises the patient to a. never miss a daily dose of thyroid replacement therapy b. avoid regular exercises until thyroid function is normalized c. use warm saltwater gargles several times a day to relieve throat pain d. reduce caloric intake by at least half the amount taken before surgery

D- With the decrease in thyroid hormone postoperatively, calories need to be reduced substantially to prevent weight gain. When a patient has had a subtotal thyroidectomy, thyroid replacement therapy is not given because exogenous hormone inhibits pituitary production of TSH and delays or prevents the restoration of thyroid tissue regeneration. Regular exercise stimulates the thyroid gland and is encouraged. Saltwater gargles are use for dryness or irritation of the mouth and throat following radioactive iodine therapy.

The nurse is caring for an 18-year-old female patient with acute lymphocytic leukemia who is scheduled to receive hematopoietic stem cell transplantation (HSCT). Which statement, if made by the patient, indicates a correct understanding of the procedure? A. "After the transplant I will feel better and can go home in 5 to 7 days." B. "I understand the transplant procedure has no dangerous side effects." C. "My brother will be a 100% match for the cells used during the transplant." D. "Before the transplant I will have chemotherapy and possibly full body radiation."

D. "Before the transplant I will have chemotherapy and possibly full body radiation." Hematopoietic stem cell transplantation (HSCT) requires eradication of diseased or cancer cells. This is accomplished by administering higher-than-usual dosages of chemotherapy with or without radiation therapy. A relative such as a brother would not be a perfect match with human leukocyte antigens

Which item would be most beneficial when providing oral care to a patient with metastatic cancer who is at risk for oral tissue injury secondary to chemotherapy? A. Firm-bristle toothbrush B. Hydrogen peroxide rinse C. Alcohol-based mouthwash D. 1 tsp salt in 1 L water mouth rinse

D. 1 tsp salt in 1 L water mouth rinse A salt-water mouth rinse will not cause further irritation to oral tissue that is fragile because of mucositis, which is a side effect of chemotherapy. A soft-bristle toothbrush will be used. One teaspoon of sodium bicarbonate may be added to the salt-water solution to decrease odor, alleviate pain, and dissolve mucin. Hydrogen peroxide and alcohol-based mouthwash are not used because they would damage the oral tissue.

The patient is receiving biologic and targeted therapy for ovarian cancer. What medication should the nurse expect to administer before therapy to combat the most common side effects of these medications? A. Morphine sulfate B. Ibuprofen (Advil) C. Ondansetron (Zofran) D. Acetaminophen (Tylenol)

D. Acetaminophen (Tylenol) Acetaminophen is administered before therapy and every 4 hours to prevent or decrease the intensity of the severe flu-like symptoms, especially with interferon which is frequently used for ovarian cancer. Morphine sulfate and ibuprofen will not decrease flu-like symptoms. Ondansetron is an antiemetic, but not used first to combat flu-like symptoms of headache, fever, chills, myalgias, etc.

The nurse is caring for a patient suffering from anorexia secondary to chemotherapy. Which strategy would be most appropriate for the nurse to use to increase the patient's nutritional intake? A. Increase intake of liquids at mealtime to stimulate the appetite. B. Serve three large meals per day plus snacks between each meal. C. Avoid the use of liquid protein supplements to encourage eating at mealtime. D. Add items such as skim milk powder, cheese, honey, or peanut butter to selected foods.

D. Add items such as skim milk powder, cheese, honey, or peanut butter to selected foods. The nurse can increase the nutritional density of foods by adding items high in protein and/or calories (such as peanut butter, skim milk powder, cheese, honey, or brown sugar) to foods the patient will eat. Increasing fluid intake at mealtime fills the stomach with fluid and decreases the desire to eat. Small frequent meals are best tolerated. Supplements can be helpful.

The patient has been diagnosed with non-small cell lung cancer. Which type of targeted therapy will most likely be used for this patient to suppress cell proliferation and promote programmed tumor cell death? A. Proteasome inhibitors B. BCR-ABL tyrosine kinase inhibitors C. CD20 monoclonal antibodies (MoAb) D. Epidermal growth factor receptor-tyrosine kinase inhibitors (EGFR-TK)

D. Epidermal growth factor receptor-tyrosine kinase inhibitors (EGFR-TK) Targeted therapies are more selective for specific molecular targets. Thus they are able to kill cancer cells with less damage to normal cells than with chemotherapy. Epidermal growth factor receptor (EGFR) is a transmembrane molecule that works through activation of intracellular tyrosine kinase (TK) to suppress cell proliferation and promote apoptosis of non-small cell lung cancer and some colorectal, head and neck, and metastatic breast cancers. Proteasome inhibitors promote accumulation of proteins that promote tumor cell death for multiple myeloma. BCR-ABL tyrosine kinase inhibitors target specific oncogenes for chronic myeloid leukemia and some GI stromal tumors. CD20 monoclonal antibodies (MoAb) bind with CD20 antigen causing cytotoxicity in non-Hodgkin's lymphoma and chronic lymphocytic leukemia.

A patient has been diagnosed with Burkitt's lymphoma. In the initiation stage of cancer, the cells genetic structure is mutated. Exposure to what may have functioned as a carcinogen for this patient? A. Bacteria B. Sun exposure C. Most chemicals D. Epstein-Barr virus

D. Epstein-Barr virus Burkitt's lymphoma consistently shows evidence of the presence of Epstein-Barr virus in vitro. Bacteria do not initiate cancer. Sun exposure causes cell alterations leading to melanoma and squamous and basal cell skin carcinoma. Long-term exposure to certain chemicals (e.g., ethylene oxide, chloroform, benzene) is known to initiate cancer.

When caring for the patient with cancer, what does the nurse understand as the response of the immune system to antigens of the malignant cells? A. Metastasis B. Tumor angiogenesis C. Immunologic escape D. Immunologic surveillance

D. Immunologic surveillance Immunologic surveillance is the process where lymphocytes check cell surface antigens and detect and destroy cells with abnormal or altered antigenic determinants to prevent these cells from developing into clinically detectable tumors. Metastasis is increased growth rate of the tumor, increased invasiveness, and spread of the cancer to a distant site in the progression stage of cancer development. Tumor angiogenesis is the process of blood vessels forming within the tumor itself. Immunologic escape is the cancer cells' evasion of immunologic surveillance that allows the cancer cells to reproduce.

The care plan for a patient with aplastic anemia should include activities to minimize the risk for which complications? A. Dyspnea and pain B. Diarrhea and fatigue C. Nausea and malnutrition D. Infection and hemorrhage

D. Infection and hemorrhage You must assist the patient in reducing infection risk. The patient is susceptible to infection and is at risk for septic shock and death. Even a low-grade temperature (>100.4° F) should be considered a medical emergency. Thrombocytopenia manifests as a predisposition to bleeding evidenced by petechiae, ecchymosis, and epistaxis. Pain is not experienced nor is diarrhea. Nausea and malnutrition are not related to this disease except as a by-product of infection. Reference: 671

Which sign or symptom would you recognize as a unique characteristic specific to hemolytic anemia? A. Tachycardia B. Weakness C. Decreased RBCs D. Jaundice

D. Jaundice Jaundice is likely because the increased destruction of RBCs causes an elevation in bilirubin levels. The spleen and liver may enlarge because of their hyperactivity, which is related to macrophage phagocytosis of the defective erythrocytes. The other symptoms are common to all types of anemia. Reference: 672

Which nursing diagnosis is most appropriate for a patient experiencing myelosuppression secondary to chemotherapy for cancer treatment? A. Acute pain B. Hypothermia C. Powerlessness D. Risk for infection

D. Risk for infection Myelosuppression is accompanied by a high risk of infection and sepsis. Hypothermia, powerlessness, and acute pain are also possible nursing diagnoses for patients undergoing chemotherapy, but the threat of infection is paramount.

A 70-year-old man who has end-stage lung cancer is admitted to the hospital with confusion and oliguria for 2 days. Which finding would the nurse report immediately to the health care provider? A. Weight gain of 2 lb B. Urine specific gravity of 1.015 C. Blood urea nitrogen of 20 mg/dL D. Serum sodium level of 118 mEq/L

D. Serum sodium level of 118 mEq/L Lung cancer cells are able to manufacture and release antidiuretic hormone (ADH) with resultant water retention and hyponatremia. Hyponatremia (serum sodium levels less than 135 mEq/L) may lead to central nervous system symptoms such as confusion, seizures, coma, and death. A weight gain may be due to fluid retention. The urine specific gravity and blood urea nitrogen are normal.

The patient is being treated with brachytherapy for cervical cancer. What factors must the nurse be aware of to protect herself when caring for this patient? A. The medications the patient is taking B. The nutritional supplements that will help the patient C. How much time is needed to provide the patient's care D. The time the nurse spends at what distance from the patient

D. The time the nurse spends at what distance from the patient The principles of ALARA (as low as reasonably achievable) and time, distance, and shielding are essential to maintain the nurse's safety when the patient is a source of internal radiation. The patient's medications, nutritional supplements, and time needed to complete care will not protect the nurse caring for a patient with brachytherapy for cervical cancer.

You anticipate that a patient with von Willebrand's disease undergoing surgery will be treated with administration of von Willebrand factor (vWF) and A. thrombin. B. factor VI. C. factor VII. D. factor VIII.

D. factor VIII. Von Willebrand's disease involves a deficiency of the von Willebrand coagulation protein, variable factor VIII deficiencies, and platelet dysfunction. Treatment includes administration of vWF and factor VIII. Reference: 684

Diagnostic testing of a patient with Grave's disease will reveal

Decreased TSH level

Type 1 Diabetes Mellitus - Nutrition therapy

Essential

Type 2 Diabetes Mellitus -

Essential

Type 1 Diabetes Mellitus - Vascular and neurologic complications

Frequent

Type 2 Diabetes Mellitus - Vascular and neurologic complications

Frequent

Type 2 Diabetes Mellitus - Symptoms

Frequently none. Fatigue, recurrent infections. May also experience polyuria, polydipsia, and polyphagia.

Type 2 Diabetes Mellitus - Nutritional status

Frequently overweight or obese. May be normal.

Enlarged thyroid gland

Goiter

Exopthalmus may occur in

Grave's disease

The two most common forms of hyperthyroidism

Grave's disease and toxic nodular goiters

Autoimmune fibrous and lymphocytic replacement of thyroid gland

Hashimoto's thyroiditis

Somogyi effect.

Hyperglycemia in the morning may be due to the Somogyi effect - A high dose of insulin produces a decline in blood glucose levels during the night. - As a result, counterregulatory hormones (e.g., glucagon, epinephrine, growth hormone, cortisol) are released, stimulating lipolysis, gluconeogenesis, and glycogenolysis, which in turn produce rebound hyperglycemia. - The danger of this effect is that when blood glucose levels are measured in the morning, hyperglycemia is apparent and the patient (or the HCP) may increase the insulin dose. - If a patient is experiencing morning hyperglycemia, checking blood glucose levels between 2:00 AM and 4:00 AM for hypoglycemia will help determine if the cause is the Somogyi effect. - The patient may report headaches on awakening and may recall having night sweats or nightmares. - A bedtime snack, a reduction in the dose of insulin, or both can help to prevent the Somogyi effect.

Type 2 Diabetes Mellitus - Endogenous insulin

Initially increased in response to insulin resistance. Secretion diminishes over time.

Type 2 Diabetes Mellitus - Type of onset

Insidious, but may go undiagnosed for years

Type 2 Diabetes Mellitus - Primary defect

Insulin resistance, decreased insulin production over time, and alterations in production of adipokines

40 The patient has a serum iron level of 120 mcg/dL and total iron binding capacity is 300 mcg/dL. The formula to calculate transferrin saturation level = (serum iron level/TIBC) x 100 = (120/300) x 100 = 40%. Text Reference - p. 628

Laboratory reports indicate that a patient has a serum iron level of 120 mcg/dL and total iron binding capacity is 300 mcg/dL. What is the transferrin saturation level of the patient? Record your answer using a whole number. _________%

Type 2 Diabetes Mellitus - Age of onset

More common in adults but can occur at any age - Incidence is increasing in children

Type 1 Diabetes Mellitus - Age of onset

More common in young people but can occur at any age

Type 2 Diabetes Mellitus - Environmental factors

Obesity, lack of exercise

Type 1 Diabetes Mellitus - Islet cell antibodies

Often present at onset

Type 1 Diabetes Mellitus - Symptoms

Polydipsia, polyuria, polyphagia, fatigue, weight loss without trying

Decreases release of thyroid hormone

Potassium iodide

Used to decrease size and vascularity of thyroid gland preoperatively

Potassium iodide

Type 1 Diabetes Mellitus - Ketosis

Prone atones or during insulin deficiency

Used to control sympathetic symptoms of hyperthyroidism

Propanolol (Inderal)

Blocks peripheral conversion of T4 to T3

Propylthiouracil (PTU)

Often used with iodine to produce euthyroid before surgery

Propylthiouracil (PTU)

Decreases thyroid secretion by damaging thyroid gland

Radioactive Iodine (131I)

Often causes hypothyroidism over time

Radioactive Iodine (131I)

Treatment of choice for hyperthyroidism in nonpregnant adults

Radioactive iodine (131I)

Drug therapy - Insulin Types

Rapid-acting insulin - lispro (Humalog) - aspart (NovoLog) - glulisine (Apidra) Short-acting insulin - regular (Humulin R, Novolin R) Intermediate-acting insulin - glargine (Lantus) - detemir (Levemir) - degludec (Tresiba) Combination therapy (premixed Inhaled insulin - Afrezza

Type 1 Diabetes Mellitus - Insulin

Required for all

Type 2 Diabetes Mellitus - Insulin

Required for some. Disease is progressive and insulin treatment may need to be added to the treatment regimen.

Type 2 Diabetes Mellitus - Ketosis

Resistant except during infection or stress

2 An increase in the neutrophil count most commonly occurs in response to infection or inflammation. Hypoxemia and coagulation do not affect directly neutrophil production. Text Reference - p. 627

Results of a patient's most recent blood work indicate an elevated neutrophil level. The nurse should recognize that this diagnostic finding most likely suggests which problem? 1 Hypoxemia 2 An infection 3 A risk of hypocoagulation 4 An acute thrombotic event

Type 1 Diabetes Mellitus - Type of onset

Signs and symptoms are usually abrupt, but disease process may be present for years

Lymphocytic infiltration of thyroid gland that may occur postpartum

Silent thyroiditis

Viral induced hypothyroidism

Subacute granulomatous thyroiditis

Sulfonylureas

Sulfonylureas include glipizide (Glucotrol, Glucotrol XL), glyburide (DiaBeta, Glynase), and glimepiride (Amaryl). - The primary action of the sulfonylureas is to increase insulin production by the pancreas. - Therefore hypoglycemia is the major side effect with sulfonylureas.

Dawn Phenomoenon

The Dawn phenomenon is also characterized by hyperglycemia that is present on awakening. - Two counterregulatory hormones (growth hormone and cortisol) which are excreted in increased amounts in the early morning hours may be the cause of this phenomenon. - The Dawn phenomenon affects a majority of people with diabetes and tends to be most severe when growth hormone is at its peak in adolescence and young adulthood. - The treatment for Dawn phenomenon is an increase in insulin or an adjustment in administration time. - Ask the patient to measure and document bedtime, nighttime (between 2:00 and 4:00 AM), and morning fasting blood glucose levels on several occasions. - If the predawn levels are less than 60 mg/dL (3.3 mmol/L) and signs and symptoms of hypoglycemia are present, the insulin dosage should be reduced. - If the 2:00 to 4:00 AM blood glucose level is high, the insulin dosage should be increased. In addition, counsel the patient on appropriate bedtime snacks.

3 The normal range for hemoglobin is 11 to16 g/dL in females and 13.2 to 17.3 mg/dL in males. Hemoglobin of 9 g/dL indicates anemia. The normal range of serum iron is 50 to 175 mcg/dL. The patient has 40 mcg/dL of serum iron, which indicates iron-deficiency anemia. The values of indirect bilirubin, folic acid, and vitamin B12 are within the normal range. The transferrin saturation is decreased in iron-deficiency anemia. The normal range of transferrin saturation is 15 to 50% but the patient has low transferrin saturation of 10% because of iron-deficiency anemia. A positive Coombs test can be observed in the patient with hemolytic anemia. The normal range for homocysteine is 3.7 to 10.4 μmol/L in females and 5.2 to 12.9 μmol/L in males. High homocysteine of 15 μmol/L is observed in patients with folic acid and cobalamin deficiency. A normal range for methylmalonic acid is <0.2 μmol/L. An increased methylmalonic acid level of 0.3 μmol/L is observed in patients with cobalamin deficiency. Text Reference - p. 628

The laboratory reports of a patient reveal a hemoglobin level of 9 mg/dL, a serum iron level of 40 mcg/dL, an indirect bilirubin level of 1 mg/dL, a vitamin B12 level of 250 pg/mL, and a folic acid level of 14 ng/mL. Which other finding will the nurse observe in the patient's report? 1 Positive coombs test 2 Homocysteine 15 μmol/L 3 Transferrin saturation of 10% 4 Methylmalonic acid 0.3 μmol/L

1 The effects of physiologic aging on clotting studies are increased D-dimer, increased erythrocyte sedimentation rate (ESR), increased fibrinogen and decreased activated partial thromboplastin time (aPTT). The normal value of D-dimer is less than 250 ng/mL. The patient has a D-dimer value of 260 ng/mL, which is an increased level. The patient's other lab results are normal. The normal value of fibrinogen is 200 to 400 mg/dL. The normal ESR value is less than 30 mm/hour. The normal aPTT is 25 to 35 seconds. Text Reference - p. 619

The nurse assesses the laboratory report of an elderly patient. Which finding is abnormal due to the aging process as recognized by the nurse? 1 D-dimer of 260 ng/mL 2 Fibrinogen of 300 mg/dL 3 Erythrocyte sedimentation rate (ESR) of 20 mm/hour 4 Activate partial thromboplastin time (aPTT) of 30 second

hence, the patient will have swollen blood vessels resulting in spider nevus. A patient with a bleeding disorder will have reduced platelet count because blood coagulation is reduced. An increase in thyroid hormone levels increases the body's metabolism. However, it does not result in spider nevus or the dilation of blood vessels. A patient with an infection will have an increase in the white blood cell count. Text Reference - p. 623

The nurse is assessing a patient with a hematologic disorder and finds a red central portion with branching radiations on the patient's face, neck, and chest. Which laboratory finding does the nurse associate with the patient's condition? 1 Elevated estrogen level 2 Elevated platelet count 3 Elevated thyroid hormone level 4 Elevated white blood cell count

3 Neutropenia is a condition in which the absolute neutrophil count (ANC) is less than 1000 cells/μL. The absolute neutrophil count is determined by multiplying the total WBC count by the percentage of neutrophils. The ANC of a patient with WBC of 9000/μL and 10% neutrophils is 900 cells/μL. Therefore, the nurse suspects neutropenia in this patient. The ANC of a patient who has a WBC of 5000/μL and 30% neutrophils is 1500 cells/μL, which does not indicate neutropenia. The ANC of a patient with a WBC of 7000/μL and 30% neutrophils is 2100 cells/μL, which is normal. Therefore, the nurse does not suspect neutropenia in this patient. The ANC of patient who has a WBC of 10,000/μL and 10% neutrophils is 2000 cells/ μL, which is normal and does not indicate neutropenia. Test-Taking Tip: You have at least a 25% chance of selecting the correct response in multiple-choice items. If you are uncertain about a question, eliminate the choices that you believe are wrong and then call on your knowledge, skills, and abilities to choose from the remaining responses. Text Reference - p. 626

The nurse is assessing four patients in a clinical care setting. Which patient does the nurse suspect has neutropenia? 1 A patient with a WBC of 5000/μL and 30% neutrophils 2 A patient with a WBC of 7000/μL and 30% neutrophils 3 A patient with a WBC of 9000/μL and 10% neutrophils 4 A patient with a WBC of 10,000/μL and 10% neutrophils

3 The normal value for MCV is 80 to 100 fL and the normal value for MCH is 27 to 34 pg. Microcytosis is a condition in which red blood cells are unusually small. Low MCV or low MCH values indicate microcytosis or hypochromia. Therefore, the red blood cell indices of patient 3 with an MCH value of 25 pg and MCV value of 76 fL indicate microcytosis. Patient 1 has an MCH of 27 pg and MCV of 85 fL, which are within the normal range and do not indicate microcytosis. Patient 2 has an MCH of 30 pg and MCV of 80 fL, which are within the normal range. Patient 4 has an MCH of 40 pg and MCV of 105 fL, higher than the normal range. Therefore, this indicates macrocytosis. Text Reference - p. 626

The nurse is assessing laboratory reports that include the red blood cell indices of four patients in a clinical care setting. Which patient's red blood cell indices reflect microcytosis? 1 Patient 1 2 Patient 2 3 Patient 3 4 Patient 4

2 An increase in neutrophils in the blood is a common diagnostic indicator of infection and tissue injury. An increase in neutrophils is not seen with inflammation, allergic reaction, or cancer. Text Reference - p. 627

The nurse is assessing the laboratory work of an individual and notes an elevated neutrophil level in the blood. The nurse suspects that possibly the patient is experiencing which problem? 1 Cancer 2 Infection 3 Inflammation 4 Allergic reaction

2 A primary function of eosinophils is to engulf antigen-antibody complexes formed during an allergic response. Basophils show response in inflammation and allergic reactions. They respond by releasing substances within the granules. Thrombocytes initiate the clotting process by producing an initial platelet plug in the early phases of the process. The main function of granulocytes is phagocytosis. Text Reference - p. 615

The nurse is caring for a patient diagnosed with an allergy. Which cells in the body may have compromised functioning? 1 Basophils 2 Eosinophils 3 Thrombocytes 4 Granulocytes

1, 2, 3 Multiple myeloma is cancer of the bone marrow. A bone scan is used to evaluate the structures of bones in patients with multiple myeloma. A skeletal x-ray is used to detect lytic lesions associated with multiple myeloma. Bence Jones protein is found in patients with multiple myeloma. A negative finding is considered normal. Computed tomography is a noninvasive radiologic examination using a computer-assisted x-ray to evaluate the lymph nodes. Magnetic resonance imaging is a procedure that produces sensitive images of soft tissue. It is used to evaluate spleen, liver, and lymph nodes. Text Reference - p. 629

The nurse is caring for a patient suspected to have multiple myeloma. Which diagnostic tests does the nurse suspect to be beneficial for this patient? Select all that apply. 1 Bone scan 2 Skeletal x-ray 3 Bence Jones protein 4 Computed tomography 5 Magnetic resonance imaging

2 ABO incompatibilities result from hemolysis of the RBCs. RBCs agglutinate when a serum antibody is present to react with the antigen on the RBC membrane. Patient 2 from blood group B has a B antigen on RBCs and anti-A serum antibodies. Agglutination occurs in the blood of this patient if he or she receives a blood transfusion from a donor from blood group A or AB. Patient 1 from blood group A has the A antigen on the RBCs and anti-B antibodies in the serum. Therefore, a blood group A patient can receive blood only from donors from group A or O. Patient 3 with AB blood group has both A and B antigens on the RBCs and does not have anti-A or anti-B antibodies. Therefore, the patient from the AB blood group can receive blood from any group. A patient from the AB blood group is called a universal recipient. Patient 4 from the O blood group does not have either A or B antigens on the surface of RBCs but has anti-A and anti-B antibodies in the serum. Therefore, patient 4 can receive blood from only from a group O donor. Text Reference - p. 626

The nurse is caring for four patients receiving blood transfusions. Which patient is at risk for agglutination? 1 Patient 1 2 Patient 2 3 Patient 3 4 Patient 4

1, 3, 5 When taking a family history, it is important to explore hematologic conditions such as anemia, hemophilia (a bleeding disorder), and other clotting disorders. Information about influenza and conjunctivitis is not relevant to hematologic conditions. Text Reference - p. 620

The nurse is obtaining the family history of a patient. Which conditions should the nurse inquire about in order to obtain information regarding existing hematologic conditions? Select all that apply. 1 Anemia 2 Influenza 3 Hemophilia 4 Conjunctivitis 5 Clotting disorders

1 As a result of the aging process, hematologic values may change but are considered normal for the older adult. The serum iron level may be decreased. Ferratin levels are increased, total iron-binding capacity may be decreased, and ESR is increased. Text Reference - p. 619

The nurse is reviewing the hematologic study report of a 70-year-old patient. Which finding should the nurse consider normal for this patient? 1 Decreased serum iron 2 Decreased ferritin 3 Increased total iron-binding capacity 4 Decreased erythrocyte sedimentation rate (ESR)

2 ANC neutropenia indicates an increased risk of developing infection. Platelet count, total RBC count, and MCHC are in the normal range. Text Reference - p. 627

The nurse is reviewing the hematologic test results of a patient with leukemia. Which test result places the patient at high risk for developing infection? 1 Platelet count 150,000/μL 2 Absolute neutrophil count (ANC) 500 cells/μL 3 Total red blood cell (RBC) count 4.3 × 106/μL 4 Mean corpuscular hemoglobin concentration (MCHC) 35%

2 Eosinophils are granulocytes that phagocytize antigen-antibody complexes formed during an allergic response. The normal eosinophil count is 2% to 4% of all white blood cells (WBCs). The dry cough, lymphocyte result, and acetaminophen use do not indicate allergies. Text Reference - p. 616

The nurse is reviewing the objective data from a patient with suspected allergies. Which assessment finding does the nurse know indicates allergies? 1 Dry cough 2 Eosinophil result 3 Lymphocyte result 4 Acetaminophen use

larks do better with morning lectures and day clinical sections. Text Reference - p. 627

The nurse is taking care of a female patient who has the following complete blood count results. Which result would cause the most concern? Refer to the chart. 1 The patient is neutropenic. Correct2 The patient may have an infection. 3 The patient is at risk for bleeding. 4 The patient is at fall risk because of anemia

2 The mean corpuscular hemoglobin concentration (MCHC) is 27%, which is lower than the normal range. The normal range of the mean corpuscular hemoglobin concentration is 32% to 37%. A decrease in this is termed hypochromia. Macrocytosis is a condition in which the mean corpuscular hemoglobin level is high. Spherocytosis is a condition in which the corpuscular hemoglobin concentration increases. Erythrocytosis is a condition in which the proportion of blood volume occupied by the red blood cells increases. It implies that the blood contains a greater number of red blood cells. Text Reference - p. 625

The nurse reviews a laboratory report of a patient that shows a mean corpuscular hemoglobin concentration (MCHC) of 27%. In which condition is the MCHC lower than the normal range? 1 Macrocytosis 2 Hypochromia 3 Spherocytosis 4 Erythrocytosis

1 Neutrophils are granular leukocytes that are involved in acute inflammatory responses. They engulf pathogens and die in one to two days. If there is an infection or sepsis, an increase in neutrophil count acts as a common diagnostic indicator. Therefore, Patient 1, with postoperative sepsis, would have an increased neutrophil count. Eosinophils are granular leukocytes that engulf antigen-antibody complexes formed during an allergic response. An increase in eosinophil count is seen in patients with neoplastic disorders such as Hodgkin's lymphoma, dermatitis, and connective tissue disorder. Text Reference - p. 616

The nurse reviews the laboratory reports of four patients. Which patient displays a common diagnostic indicator by an increase in the neutrophil count? 1 Patient 1 2 Patient 2 3 Patient 3 4 Patient 4

1 Lymphedema is the accumulation of lymphatic fluid due to an obstruction in the lymphatic flow or system. A mastectomy (the surgical removal of breast tissue) also involves the dissection of the axillary lymph nodes. This can obstruct the lymphatic flow, which can lead to lymphedema. Therefore, Patient 1 is at risk for developing lymphedema. Gastrectomies, splenectomies, and hepatectomies do not involve the resectioning or dissection of lymph nodes. Test-Taking Tip: The lymphatic system consists of lymph fluid, lymphatic capillaries, ducts, and lymph nodes that carry fluid from the interstitial spaces to the blood. This helps in selecting the correct answer. Text Reference - p. 618

The nurse reviews the medical records of four patients. Which patient is at risk for an alteration in the hematologic system that could lead to lymphedema? 1 Patient 1 2 Patient 2 3 Patient 3 4 Patient 4

1, 3, 4 Increased homocysteine and MMA, along with decreased cobalamin, indicate cobalamin deficiency, which is a nutrient needed for RBC production. Decreased reticulocytes indicate low bone marrow activity in producing RBCs, not available nutrients. Elevated ESR is related to an increased inflammatory process, not anemia. Text Reference - p. 616

The patient has anemia and has had laboratory tests done to diagnose the cause. Which results should the nurse know indicate a lack of nutrients needed to produce new red blood cells (RBCs)? Select all that apply. 1 Increased homocysteine 2 Decreased reticulocyte count 3 Decreased cobalamin (vitamin B12) 4 Increased methylmalonic acid (MMA) 5 Elevated erythrocyte sedimentation rate (ESR)

thus, this patient will not be in the prone position. The analgesic should have been administered preprocedure. Metal objects would be removed for a magnetic resonance imaging (MRI), not a bone marrow biopsy. Text Reference - p. 630

The thrombocytopenic patient has had a bone marrow biopsy taken from the posterior iliac crest. What nursing care is the priority for this patient after this procedure? 1 Position the patient prone 2 Apply a pressure dressing 3 Administer analgesic for pain 4 Return metal objects to the patien

Type 1 Diabetes Mellitus - Nutritional status

Thin, normal, or obese

Malignant of benign deformity of the thyroid gland

Thyroid nodules

Signs and Symptoms of Hypoglycemia - tachycardia - tachypnea - diaphoresis - blurred vision/double vision - weakness - palpations - confusion - tremors/shaking - irritability - hunger - pallor - numbness of tongue and lips

Treatment - provide rapidly absorbed source of glucose - fruit juice/ cola - graham crackers - sugar cubes - hard candy - provide regular meal as symptoms improve

Signs and Symptoms of Hyperglycemia - nausea - vomiting - lethargy - thirst - increased temperature - flushed/dry skin - poor skin tumor - fatigue - irritability - numbness - tachycardia - coma - fruity breath

Treatment - restore fluid balance - replace electrolytes - lower blood glucose with regular insulin - monitor consciousness and vital signs, INO, electrolytes - provide emotional support

Type 1 Diabetes Mellitus - Etiology and Pathophysiology

Type 1 diabetes is an autoimmune disorder, in which the body develops antibodies against insulin and/or the pancreatic β-cells that produce insulin. This eventually results in not enough insulin for a person to survive. Autoantibodies to the islet cells cause a reduction of 80% to 90% of normal function before hyperglycemia and other manifestations occur.

Drug therapy - Insulin Regimens Once a day Single dose

Type of Insulin: Intermediate (NPH) At bedtime - One injection should provide nighttime coverage. OR Long-acting (glargine [Lantus], detemir [Levemir]) In AM or at bedtime - One injection may last up to 24 hr with less defined peaks and less chance for hypoglycemia. Does not cover postprandial blood glucose levels.

Drug therapy - Insulin Regimens Three times a day Combination of mixed and single dose

Type of Insulin: NPH and regular or rapid (both regular and rapid are shown on the diagram) Before breakfast - Regular or rapid Before dinner - NPH At bedtime - Three injections provide coverage for 24 hr, particularly during early AM hours. Decreased potential for 2-3 AM hypoglycemia.

Drug therapy - Insulin Regimens Twice a day Split-mixed dose

Type of Insulin: NPH and regular or rapid (both regular and rapid are shown on the diagram) Before breakfast and at dinner - Two injections provide coverage for 24 hr. Patient must eat at certain times to avoid hypoglycemia.

Drug therapy - Insulin Regimens Basal-bolus Multiple dose

Type of Insulin: Regular or rapid (both regular and rapid are shown on the diagram) Before breakfast, lunch, and dinner - Long-acting (glargine or detemir) Once or twice a day OR Regular or rapid (both regular and rapid are shown on the diagram) Before breakfast, lunch, and dinner - NPH Twice a day - More flexibility is allowed at mealtimes and for amount of food intake. Good postprandial coverage. Preprandial blood glucose checks and establishing and following individualized algorithms are necessary. Patients with type 1 diabetes require basal insulin to cover 24 hr. Most physiologic approach, except for pump.

Type 1 Diabetes Mellitus - Environmental factors

Virus, toxins

they constitute about 50 percent to 70 percent of the white blood cells. Text Reference - p. 615

What is the percentage of lymphocytes in the blood? 1 2 percent to 4 percent 2 4 percent to 8 percent 3 20% to 40% 4 50% to 70%

decreased urine output may mean some abnormality and needs to be noted. Offensive sweating and excessive hunger and thirst are not related to elimination patterns in hematologic conditions. Test-Taking Tip: Be alert for details. Details provided in the stem of the item, such as behavioral changes or clinical changes (or both) within a certain time period, can provide a clue to the most appropriate response or, in some cases, responses. Text Reference - p. 621

When assessing a patient's elimination patterns to determine the presence of hematologic conditions, the nurse should note which findings as important. Select all that apply. 1 Offensive sweating 2 Dark-colored urine 3 Black-colored stools 4 Decreased urine output 5 Excessive hunger and thirst

3 Hemoglobin levels begin to decrease in both men and women after middle age, with low normal levels seen in most older people. Platelet levels are unchanged. The total white blood cell (WBC) count and differential generally are not affected by aging. The ESR is increased with aging. Text Reference - p. 619

When reviewing laboratory results of an 83-year-old patient who came to the clinic for annual blood work, the nurse would expect to find 1 Decreased platelet count 2 Increased leukocyte count 3 Decreased hemoglobin levels 4 Decreased erythrocyte sedimentation rate (ESR)

2 Manufactured in the liver, fibrinogen (also known as clotting factor I) acts as a source of fibrin and helps to form clots. Calcium (coagulation factor IV) is required at several points in the coagulation cascade. Known as coagulation factor II, prothrombin is converted to thrombin, and activates fibrinogen. Proaccelerin is coagulation factor V, which binds with coagulation factor X to activate prothrombin. Text Reference - p. 618

Which blood coagulation factor acts as a source to form clot? 1 Calcium 2 Fibrinogen 3 Prothrombin 4 Proaccelerin

they ingest dead cells, tissue debris, and defective red blood cells. Basophils stimulate antigens in response to tissue injury. Eosinophils activate in response to allergies and engulf antigen-antibody complex. Thrombocytes (platelets) help in clot formation. Text Reference - p. 616

Which cells ingest dead cells, tissue debris, and defective red blood cells? 1 Basophils 2 Eosinophils 3 Monocytes 4 Thrombocytes

it increases adhesiveness. Test-Taking Tip: Blood clotting is an important process that prevents excessive bleeding when a blood vessel is injured. Read carefully to answer the question. Text Reference - p. 617

Which component forms an adhesive bridge between platelets and vascular subendothelial structures in the clotting process? 1 Serotonin 2 Platelet factor 3 3 Von Willebrand factor 4 Adenosine diphosphate

it helps degrade plasminogen. Test-Taking Tip: You have at least a 25 percent chance of selecting the correct response in multiple-choice items. If you are uncertain about a question, eliminate the choices that you believe are wrong and then call on your knowledge, skills, and abilities to choose from the remaining responses. Text Reference - p. 618

Which component in the picture represents the process in which the binding of plasmin results in degradation of a clot? 1 Plasmin 2 Fibrin clot 3 Fibrinolysis 4 Plasminogen activator

4 Hemoglobin acts as a buffer that maintains blood acid-base balance. Platelets promote blood coagulation. Albumin is a plasma protein that helps to maintain oncotic pressure of blood. Leukocytes combat the invasion of pathogens. Text Reference - p. 615

Which component of the blood helps maintain acid-base balance? 1 Platelets 2 Albumin 3 Leukocytes 4 Hemoglobin

it starts the intrinsic pathway to activate clot formation. Test-Taking Tip: Make certain that the answer you select is reasonable and obtainable under ordinary circumstances and that the action can be carried out in the given situation. Text Reference - p. 618

Which components play a major role in the last phase of hemostasis? Select all that apply. 1 Protein C 2 Protein S 3 Proconvertin 4 Antithrombin III 5 Hageman factor

4 Hematemesis is bright red, brown, or black vomitus, associated with an underlying disease like peptic ulcer disease. Thalassemia is a hereditary autosomal disorder characterized by the abnormal growth of red blood cells or hemoglobin. Sickle cell anemia is a hereditary disorder, which manifests as a distorted shape of hemoglobin. Pernicious anemia is a deficiency in the production of red blood cells due to lack of vitamin B12. Text Reference - p. 620

Which condition is the most common cause for hematemesis? 1 Thalassemia 2 Sickle cell disease 3 Pernicious anemia 4 Peptic ulcer disease

4 The effects of aging on hematologic studies include increased ferritin, increased mean cell volume (MCV), increased ESR, decreased serum iron, and decreased total iron binding capacity. A total iron binding capacity (TIBC) of 200 mcg/dL indicates a lower than normal value. The normal range of TIBC is 250 to 425 mcg/dL. The patient's ferritin, serum iron, and MCV values are normal. The normal ranges are ferritin 10 to 250 ng/mL, serum iron 50 to 175 mcg/dL, and MCV 80 to 100 fL. Test-Taking Tip: Recollecting the effects of physiological aging on the hematologic system and correlating them with normal laboratory values will help you choose the correct answer. Text Reference - p. 619

Which laboratory finding is abnormal due to the aging process? 1 Ferritin level of 200 μg/L 2 Serum iron level of 150 mcg/dL 3 Mean cell volume of (MCV) 90 fL 4 Total iron-binding capacity (TIBC) of 200 mcg/dL

the synthesis of this protein occurs in the liver. The spleen is the site for the synthesis of red blood cells during fetal development. Bone marrow is the site for maturation of red blood cells. Macrophages help to store ferritin, a storage form of iron. Text Reference - p. 616

Which organ acts as a site for the synthesis of iron carrier proteins? 1 Liver 2 Spleen 3 Bone marrow 4 Macrophages

2 Myeloblasts differentiate to form neutrophilic metamyelocytes, which further differentiate to create band cells. Band cells divides and give rise to neutrophils. Monoblasts creates monocytes. Erythroblasts differentiate to form erythrocytes, or red blood cells. Megakaryoblasts differentiate to form thrombocytes, or platelets. Text Reference - p. 614

Which stem cell differentiates to form neutrophils? 1 Monoblast 2 Myeloblast 3 Erythroblast 4 Megakaryoblast

3 A smooth, shiny tongue surface can be observed in patients with pernicious anemia. A pallor of the gingival and mucosal membrane is observed due to low hemoglobin level. Patients with neutropenia may have gingival and mucosal ulceration, swelling, or bleeding. Patients with polycythemia may have decreased visual acuity and transient episodic redness of the skin. A patient with elevated estrogen levels may have spider nevus, a form of telangiectasia. Text Reference - p. 623

While assessing a patient, the nurse finds a smooth and shiny tongue surface. What does the nurse infer from these findings? 1 The patient has neutropenia. 2 The patient has polycythemia. 3 The patient has pernicious anemia. 4 The patient has elevated estrogen levels.

2 In patients with red blood cell (RBC) disorders the skin may be pale or pasty, or it may have a cyanotic tinge in severe anemia. Yellowing of the skin, or jaundice, is caused by an accumulation of bile pigment caused by rapid or excessive hemolysis or liver damage. Flushed skin in the face and neck areas may indicate an increase in hemoglobin (polycythemia) or congestion of the capillaries. Erythrocytosis often produces small vessel occlusions, causing a purple, mottled appearance of the face, nose, fingers, or toes. Text Reference - p. 623

While assessing the skin of a patient with anemia, the nurse would look for which manifestations? 1 Yellow appearance of the skin 2 Color pale or with a cyanotic tinge 3 Flushed, red skin of the face and neck 4 Purple, mottled appearance of face, nose, or fingers

4 Methylmalonic acid (MMA) is an indirect test for cobalamin (vitamin B12) because MMA metabolism requires cobalamin. It helps to differentiate cobalamin deficiency from folic acid deficiency. The normal value of methylmalonic acid is less than 0.2 μmol/L. Therefore, a high methylmalonic acid level of 0.4 μmol/L indicates reduced metabolism due to a deficiency of cobalamin. Because the patient has a low hemoglobin level of 10 g/dL, the nurse infers that the patient has vitamin B12-deficiency anemia. The patient with hemolytic anemia will have high bilirubin levels and have a positive Coombs test. The patient with iron deficiency anemia will have decreased serum iron, ferritin, and an increased total iron binding capacity. The patient with folic acid deficiency anemia will have a value of folic acid less than 3 to16 ng/mL. Test-Taking Tip: Multiple-choice questions can be challenging, because students think that they will recognize the right answer when they see it or that the right answer will somehow stand out from the other choices. This is a dangerous misconception. The more carefully the question is constructed, the more each of the choices will seem like the correct response. Text Reference - p. 628

While reviewing the laboratory test results of a patient, the nurse finds that the patient's methylmalonic acid level is 0.4 μmol/L and hemoglobin is 10 mg/dL. What does the nurse infer from this finding? 1 The patient has hemolytic anemia. 2 The patient has iron-deficiency anemia. 3 The patient has folic acid deficiency anemia. 4 The patient has vitamin B12-deficiency anemia

insulin resistance

a condition in which body tissues do not respond to the action of insulin because insulin receptors are unresponsive, are insufficient in number, or both. - pear or apple shaped body (fat in abdomen) - may develop acanthosis nigricans

Idiopathic diabetes

a form of type 1 diabetes that is strongly inherited and not related to autoimmunity. It only occurs in a small number of people with type 1 diabetes, most often of Hispanic, African, or Asian ancestry.

Latent autoimmune diabetes in adults (LADA)

a slowly progressing autoimmune form of type 1 diabetes, occurs in adults, and is often mistaken for type 2 diabetes.

An African American woman with a history of breast cancer has panhypopituitarism from radiation therapy for primary pituitary tumors. Which medications should the nurse teach her about needing for the rest of her life (select all that apply)? a. Cortisol b. Vasopressin c. Sex hormones d. Levothyroxine (Synthroid) e. Growth hormone (somatropin [Omnitrope]) f. Dopamine agonists (bromocriptine [Parlodel])

a, b, d, e. With panhypopituitarism, lifetime hormone replacement is needed for cortisol, vasopressin, thyroid, and GH. Sex hormones will not be replaced because of the patient's history of breast cancer. Dopamine agonists will not be used because they reduce secretion of GH, which has already been achieved with the radiation.

Which characteristics describe the use of RAI (select all that apply)? a. Often causes hypothyroidism over time b. Decreases release of thyroid hormones c. Blocks peripheral conversion of T4 to T3 d. Treatment of choice in nonpregnant adults e. Decreases thyroid secretion by damaging thyroid gland f. Often used with iodine to produce euthyroid before surgery

a, d, e. RAI causes hypothyroidism over time by damaging thyroid tissue and is the treatment of choice for nonpregnant adults. Potassium iodide decreases the release of thyroid hormones and decreases the size of the thyroid gland preoperatively. Propylthiouracil (PTU) blocks peripheral conversion of T4 to T3 and may be used with iodine to produce a euthyroid state before surgery.

As a precaution for vocal cord paralysis from damage to the recurrent laryngeal nerve during thyroidectomy surgery, what equipment should be in the room in case it is needed for this emergency situation? a. Tracheostomy tray c. IV calcium gluconate b. Oxygen equipment d. Paper and pencil for communication

a. A tracheostomy tray is in the room to use if vocal cord paralysis occurs from recurrent laryngeal nerve damage or for laryngeal stridor from tetany. The oxygen equipment may be useful but will not improve oxygenation with vocal cord paralysis without a tracheostomy. IV calcium salts will be used if hypocalcemia occurs from parathyroid damage. The paper and pencil for communication may be helpful, especially if a tracheostomy is performed, but will not aid in emergency oxygenation of the patient.

A patient with acromegaly is treated with a transsphenoidal hypophysectomy. What should the nurse do postoperatively? a. Ensure that any clear nasal drainage is tested for glucose. b. Maintain the patient flat in bed to prevent cerebrospinal fluid (CSF) leakage. c. Assist the patient with toothbrushing every 4 hours to keep the surgical area clean. d. Encourage deep breathing, coughing, and turning to prevent respiratory complications.

a. A transsphenoidal hypophysectomy involves entry into the sella turcica through an incision in the upper lip and gingiva into the floor of the nose and the sphenoid sinuses. Postoperative clear nasal drainage with glucose content indicates cerebrospinal fluid (CSF) leakage from an open connection to the brain, putting the patient at risk for meningitis. After surgery, the patient is positioned with the head elevated to avoid pressure on the sella turcica. Coughing and straining are avoided to prevent increased intracranial pressure and CSF leakage. Although mouth care is required every 4 hours, toothbrushing should not be performed because injury to the suture line may occur.

Which statement accurately describes Graves' disease? a. Exophthalmos occurs in Graves' disease. b. It is an uncommon form of hyperthyroidism. c. Manifestations of hyperthyroidism occur from tissue desensitization to the sympathetic nervous system. d. Diagnostic testing in the patient with Graves' disease will reveal an increased thyroid-stimulating hormone (TSH) level.

a. Exophthalmos or protrusion of the eyeballs may occur in Graves' disease from increased fat deposits and fluid in the orbital tissues and ocular muscles, forcing the eyeballs outward. Graves' disease is the most common form of hyperthyroidism. Increased metabolic rate and sensitivity of the sympathetic nervous system lead to the clinical manifestations. Thyroid-stimulating hormone (TSH) level is decreased in Graves' disease.

When caring for a patient with primary hyperaldosteronism, the nurse would question a health care provider's prescription for which drug? a. Furosemide (Lasix) c. Spironolactone (Aldactone) b. Amiloride (Midamor) d. Aminoglutethimide (Cytadren)

a. Hyperaldosteronism is an excess of aldosterone, which is manifested by sodium and water retention and potassium excretion. Furosemide is a potassium-wasting diuretic that would increase the potassium deficiency. Aminoglutethimide blocks aldosterone synthesis. Spironolactone and amiloride are potassium-sparing diuretics.

A patient is admitted to the hospital with a diagnosis of Cushing syndrome. On physical assessment of the patient, what should the nurse expect to find? a. Hypertension, peripheral edema, and petechiae b. Weight loss, buffalo hump, and moon face with acne c. Abdominal and buttock striae, truncal obesity, and hypotension d. Anorexia, signs of dehydration, and hyperpigmentation of the skin

a. The effects of adrenocortical hormone excess, especially glucocorticoid excess, include weight gain from accumulation and redistribution of adipose tissue, sodium and water retention, glucose intolerance, protein wasting, loss of bone structure, loss of collagen, and capillary fragility leading to petechiae. Clinical manifestations of adrenocortical hormone deficiency include hypotension, dehydration, weight loss, and hyperpigmentation of the skin.

During care of the patient with SIADH, what should the nurse do? a. Monitor neurologic status at least every 2 hours. b. Teach the patient receiving treatment with diuretics to restrict sodium intake. c. Keep the head of the bed elevated to prevent antidiuretic hormone (ADH) release. d. Notify the health care provider if the patient's blood pressure decreases more than 20 mm Hg from baseline.

a. The patient with syndrome of inappropriate antidiuretic hormone (SIADH) has marked dilutional hyponatremia and should be monitored for decreased neurologic function and seizures every 2 hours. Sodium intake is supplemented because of the hyponatremia and sodium loss caused by diuretics. ADH release is reduced by keeping the head of the bed flat to increase left atrial filling pressure. A reduction in blood pressure (BP) indicates a reduction in total fluid volume and is an expected outcome of treatment.

What preoperative instruction should the nurse give to the patient scheduled for a subtotal thyroidectomy? a. How to support the head with the hands when turning in bed b. Coughing should be avoided to prevent pressure on the incisionm c. Head and neck will need to remain immobile until the incision heals d. Any tingling around the lips or in the fingers after surgery is expected and temporary

a. To prevent strain on the suture line postoperatively, the patient's head must be manually supported while turning and moving in bed but range-of-motion exercises for the head and neck are also taught preoperatively to be gradually implemented after surgery. There is no contraindication for coughing and deep breathing and these should be carried out postoperatively. Tingling around the lips or fingers is a sign of hypocalcemia, which may occur if the parathyroid glands are inadvertently removed during surgery. This sign should be reported immediately.

Identify the rationale for having the following items immediately available in the patient's room following thyroidectomy a. Tracheostomy tray b. Calcium salts for IV administration c. Oxygen equipment

a. To use in case airway obstruction occurs because of vocal cord paralysis from recurrent laryngeal nerve damage during surgery or laryngeal stridor occurs with tetany b. needed in case hypocalcemia occurs form parathyroid gland removal or damage during surgery, resulting in tetany c. in case of airway obstruction, laryngeal stridor or edema around trachea

A patient who is on corticosteroid therapy for treatment of an autoimmune disorder has the following additional drugs ordered. How is the need for these drugs related to the effects of corticosteroids a. Furosemide (Lasix) b. Pantoprazole (Protonix) c. Alendronate (Fosamax) d. Insulin e. Potassium

a. sodium and fluid retention because of mineralcorticoid effect b. gastrointestinal irritation with an increase in secretion of pepsin and hydrochloric acid c. corticosteroid induced osteoperosis

A patient has been diagnosed with hypoparathyroidism. What manifestations should the nurse expect to observe (select all that apply)? a. Skeletal pain b. Dry, scaly skin c. Personality changes d. Abdominal cramping e. Cardiac dysrhythmias f. Muscle spasms and stiffness

b, c, d, e, f. In hypoparathyroidism the patient has inadequate circulating parathyroid hormone (PTH) that leads to hypocalcemia from the inability to maintain serum calcium levels. With hypocalcemia there is muscle stiffness and spasms, which can lead to cardiac dysrhythmias and abdominal cramps. There can also be personality and visual changes and dry, scaly skin.

What is an appropriate nursing intervention for the patient with hyperparathyroidism? a. Pad side rails as a seizure precaution. b. Increase fluid intake to 3000 to 4000 mL daily. c. Maintain bed rest to prevent pathologic fractures. d. Monitor the patient for Trousseau's and Chvostek's signs.

b. A high fluid intake is indicated in hyperparathyroidism to dilute the hypercalcemia and flush the kidneys so that calcium stone formation is reduced. Seizures are not associated with hyperparathyroidism. Impending tetany of hypoparathyroidism after parathyroidectomy can be noted with Trousseau's and Chvostek's signs. The patient with hyperparathyroidism is at risk for pathologic fractures resulting from decreased bone density but mobility is encouraged to promote bone calcification.

During discharge teaching for the patient with Addison's disease, which statement by the patient indicates that the nurse needs to do additional teaching? ma. "I should always call the doctor if I develop vomiting or diarrhea." b. "If my weight goes down, my dosage of steroid is probably too high." c. "I should double or triple my steroid dose if I undergo rigorous physical exercise." d. "I need to carry an emergency kit with injectable hydrocortisone in case I can't take my medication by mouth."

b. A weight reduction in the patient with Addison's disease may indicate a fluid loss and a dose of replacement therapy that is too low rather than too high. Because vomiting and diarrhea are early signs of crisis and because fluid and electrolytes must be replaced, patients should notify their health care provider if these symptoms occur. Patients with Addison's disease are taught to take two to three times their usual dose of steroids if they become ill, have teeth extracted, or engage in rigorous physical activity and should always have injectable hydrocortisone available if oral doses cannot be taken.

A patient with hypothyroidism is treated with levothyroxine (Synthroid). What should the nurse include when teaching the patient about this therapy? a. Explain that alternate-day dosage may be used if side effects occur. b. Provide written instruction for all information related to the drug therapy. c. Assure the patient that a return to normal function will occur with replacement therapy. d. Inform the patient that the drug must be taken until the hormone balance is reestablished.

b. Because of the mental sluggishness, inattentiveness, and memory loss that occur with hypothyroidism, it is important to provide written instructions and repeat information when teaching the patient. Replacement therapy must be taken for life and alternate-day dosing is not therapeutic. Although most patients return to a normal state with treatment, cardiovascular conditions and psychoses may persist.

The nurse has identified the nursing diagnosis of fatigue for a patient who is hypothyroid. What should the nurse do while caring for this patient? a. Monitor for changes in orientation, cognition, and behavior. b. Monitor for vital signs and cardiac rhythm response to activity. c. Monitor bowel movement frequency, consistency, shape, volume, and color. d. Assist in developing well-balanced meal plans consistent with level of energy expenditure.

b. Cardiorespiratory response to activity is important to monitor in this patient to determine the effect of activities and plan activity increases. Monitoring changes in orientation, cognition, and behavior are interventions for impaired memory. Monitoring bowels is needed to plan care for the patient with constipation. Assisting with meal planning will help the patient with imbalanced nutrition: more than body requirements to lose weight if needed.

When caring for a patient with nephrogenic diabetes insipidus, what should the nurse expect the treatment to include? a. Fluid restriction b. Thiazide diuretics c. A high-sodium diet d. Chlorpropamide (Diabinese)

b. In nephrogenic diabetes insipidus, the kidney is unable to respond to ADH, so vasopressin or hormone analogs are not effective. Thiazide diuretics slow the glomerular filtration rate (GFR) in the kidney and produce a decrease in urine output. Low-sodium diets (<3 g/day) are also thought to decrease urine output. Fluids are not restricted because the patient could easily become dehydrated.

Priority Decision: What is the priority nursing intervention during the management of the patient with pheochromocytoma? a. Administering IV fluids c. Administering β-adrenergic blockers b. Monitoring blood pressure d. Monitoring intake and output and daily weights

b. Pheochromocytoma is a catecholamine-producing tumor of the adrenal medulla, which may cause severe, episodic hypertension

When the patient with parathyroid disease experiences symptoms of hypocalcemia, what is a measure that can be used to temporarily raise serum calcium levels? a. Administer IV normal saline. c. Administer furosemide (Lasix) as ordered. b. Have patient rebreathe in a paper bag. d. Administer oral phosphorus supplements.

b. Rebreathing in a paper bag promotes carbon dioxide retention in the blood, which lowers pH and creates an acidosis. An acidemia enhances the solubility and ionization of calcium, increasing the proportion of total body calcium available in physiologically active form and relieving the symptoms of hypocalcemia. Saline promotes calcium excretion, as does furosemide. Phosphate levels in the blood are reciprocal to calcium and an increase in phosphate promotes calcium excretion.

A patient with SIADH is treated with water restriction. What does the patient experience when the nurse determines that treatment has been effective? a. Increased urine output, decreased serum sodium, and increased urine specific gravity b. Increased urine output, increased serum sodium, and decreased urine specific gravity c. Decreased urine output, increased serum sodium, and decreased urine specific gravity d. Decreased urine output, decreased serum sodium, and increased urine specific gravity

b. The patient with SIADH has water retention with hyponatremia, decreased urine output, and concentrated urine with high specific gravity. Improvement in the patient's condition is reflected by increased urine output, normalization of serum sodium, and more water in the urine, thus decreasing the specific gravity.

What medication is used with thyrotoxicosis to block the effects of the sympathetic nervous stimulation of the thyroid hormones? a. Potassium iodide c. Propylthiouracil (PTU) b. Atenolol (Tenormin) d. Radioactive iodine (RAI)

b. The β-adrenergic blocker atenolol is used to block the sympathetic nervous system stimulation by thyroid hormones. Potassium iodide is used to prepare the patient for thyroidectomy or for treatment of thyrotoxic crisis to inhibit the synthesis of thyroid hormones. Antithyroid medications inhibit the synthesis of thyroid hormones. Radioactive iodine (RAI) therapy destroys thyroid tissue, which limits thyroid hormone secretion.

A patient with Addison's disease comes to the emergency department with complaints of nausea, vomiting, diarrhea, and fever. What collaborative care should the nurse expect? a. IV administration of vasopressors b. IV administration of hydrocortisone c. IV administration of D5W with 20 mEq KCl d. Parenteral injections of adrenocorticotropic hormone (ACTH)

b. Vomiting and diarrhea are early indicators of Addisonian crisis and fever indicates an infection, which is causing additional stress for the patient. Treatment of a crisis requires immediate glucocorticoid replacement and IV hydrocortisone, fluids, sodium, and glucose are necessary for 24 hours. Addison's disease is a primary insufficiency of the adrenal gland and adrenocorticotropic hormone (ACTH) is not effective, nor would vasopressors be effective with the fluid deficiency of Addison's disease. Potassium levels are increased in Addison's disease and KCl would be contraindicated.

The patient is diagnosed with syndrome of inappropriate antidiuretic hormone (SIADH). What manifestation should the nurse expect to find? a. Decreased body weight c. Increased plasma osmolality b. Decreased urinary output d. Increased serum sodium levels

b. With increased antidiuretic hormone (ADH), the permeability of the renal distal tubules is increased, so water is reabsorbed into circulation. Decreased output of concentrated urine with increased urine osmolality and specific gravity occur. In addition, fluid retention with weight gain, serum hypoosmolality, dilutional hyponatremia, and hypochloremia occur.

What findings are commonly found in a patient with a prolactinoma? a. Gynecomastia in men b. Profuse menstruation in women c. Excess follicle-stimulating hormone (FSH) and luteinizing hormone (LH) d. Signs of increased intracranial pressure, including headache, nausea, and vomiting

d. Compression of the optic chiasm can cause visual problems as well as signs of increased intracranial pressure, including headache, nausea, and vomiting. About 30% of prolactinomas will have excess prolactin secretion with manifestations of impotence in men, galactorrhea or amenorrhea in women without relationship to pregnancy, and decreased libido in both men and women. There is decreased follicle-stimulating hormone (FSH) and luteinizing hormone (LH).

A patient is admitted to the hospital with thyrotoxicosis. On physical assessment of the patient, what should the nurse expect to find? a. Hoarseness and laryngeal stridor b. Bulging eyeballs and dysrhythmias c. Elevated temperature and signs of heart failure d. Lethargy progressing suddenly to impairment of consciousness

c. A hyperthyroid crisis results in marked manifestations of hyperthyroidism, with severe tachycardia, heart failure, shock, hyperthermia, restlessness, irritability, abdominal pain, vomiting, diarrhea, delirium, and coma. Although exophthalmos may be present in the patient with Graves' disease, it is not a significant factor in hyperthyroid crisis. Hoarseness and laryngeal stridor are characteristic of the tetany of hypoparathyroidism and lethargy progressing to coma is characteristic of myxedema coma, a complication of hypothyroidism.

A patient suspected of having acromegaly has an elevated plasma growth hormone (GH) level. In acromegaly, what would the nurse also expect the patient's diagnostic results to indicate? a. Hyperinsulinemia b. Plasma glucose of <70 mg/dL (3.9 mmol/L) c. Decreased GH levels with an oral glucose challenge test d. Elevated levels of plasma insulin-like growth factor-1 (IGF-1)

d. A normal response to growth hormone (GH) secretion is stimulation of the liver to produce somatomedin C, or insulin-like growth factor-1 (IGF-1), which stimulates growth of bones and soft tissues. The increased levels of somatomedin C normally inhibit GH but in acromegaly the pituitary gland secretes GH despite elevated IGF-1 levels. When both GH and IGF-1 levels are increased, overproduction of GH is confirmed. GH also causes elevation of blood glucose and normally GH levels fall during an oral glucose challenge but not in acromegaly.

In a patient with central diabetes insipidus, what will the administration of ADH during a water deprivation test result in? a. Decrease in body weight c. Decrease in blood pressure b. Increase in urinary output d. Increase in urine osmolality

d. A patient with central diabetes insipidus has a deficiency of ADH with excessive loss of water from the kidney, hypovolemia, hypernatremia, and dilute urine with a low specific gravity. When vasopressin is administered, the symptoms are reversed, with water retention, decreased urinary output that increases urine osmolality, and an increase in BP.

Priority Decision: When replacement therapy is started for a patient with long-standing hypothyroidism, what is most important for the nurse to monitor the patient for? a. Insomnia c. Nervousness b. Weight loss d. Dysrhythmias

d. All these manifestations may occur with treatment of hypothyroidism. However, as a result of the effects of hypothyroidism on the cardiovascular system, when thyroid replacement therapy is started myocardial oxygen consumption is increased and the resultant oxygen demand may cause angina, cardiac dysrhythmias, and heart failure, so monitoring for dysrhythmias is most important.

A patient who is on corticosteroid therapy for treatment of an autoimmune disorder has the following additional drugs ordered. Which one is used to prevent corticosteroid-induced osteoporosis? a. Potassium c. Alendronate (Fosamax) b. Furosemide (Lasix) d. Pantoprazole (Protonix)

c. Alendronate (Fosamax) is used to prevent corticosteroid- induced osteoporosis. Potassium is used to prevent the mineralocorticoid effect of hypokalemia. Furosemide (Lasix) is used to decrease sodium and fluid retention from the mineralocorticoid effect. Pantoprazole (Protonix) is used to prevent gastrointestinal (GI) irritation from an increase in secretion of pepsin and hydrochloric acid.

Planning

(1) engage in self-care behaviors to actively manage his or her diabetes (2) experience few or no hyperglycemia or hypoglycemia emergencies (3) maintain blood glucose levels at normal or near-normal levels (4) prevent or minimize chronic complications related to diabetes (5) adjust lifestyle to accommodate the diabetes plan with a minimum of stress.

A patient is scheduled for a bilateral adrenalectomy. During the postoperative period, what should the nurse expect related to the administration of corticosteroids? a. Reduced to promote wound healing b. Withheld until symptoms of hypocortisolism appear c. Increased to promote an adequate response to the stress of surgery d. Reduced because excessive hormones are released during surgical manipulation of adrenal glands

c. Although the patient with Cushing syndrome has excess corticosteroids, removal of the glands and the stress of surgery require that high doses of corticosteroids (cortisone) be administered postoperatively for several days before weaning the dose. The nurse should monitor the patient's vital signs postoperatively to detect whether large amounts of hormones were released during surgical manipulation, obtain morning urine specimens for cortisol measurement to evaluate the effectiveness of the surgery, and provide dressing changes with aseptic technique to avoid infection as usual inflammatory responses are suppressed.

Exenatide (Byetta)

- Acute pancreatitis and kidney problems have been associated with its use.

Pramlintide (Symlin)

- Can cause severe hypoglycemia when used with insulin.

Carbohydrate counting

- Carbohydrate counting is a meal planning technique used to keep track of the amount of carbohydrates eaten with each meal and per day. - Advise patients to keep carbohydrates within a healthy range. - The amount of total carbohydrates per day depends on blood glucose levels, age, weight, activity level, patient preference, and prescribed medications. - A serving size of carbohydrates is 15 g. A typical adult usually starts with 45 to 60 g of carbohydrate per meal

Liraglutide (Victoza) and Dulaglutide (Trulicity)

- Do not use in patients with a personal or family history of medullary thyroid cancer. - Acute pancreatitis has been associated with its use.

Metformin

- Do not use in patients with kidney disease, liver disease, or heart failure. Lactic acidosis is a rare complication of metformin accumulation. - IV contrast media that contain iodine pose a risk of acute kidney injury, which could exacerbate metformin-induced lactic acidosis. - To reduce risk of kidney injury, discontinue metformin a day or two before the procedure. - May be resumed 48 hours after the procedure, assuming kidney function is normal. - Do not use in people who drink excessive amounts of alcohol. - Take with food to minimize GI side effects.

Diabetes Mellitus - Objective Data

- Eyes: Soft, sunken eyeballs.* History of vitreal hemorrhages, cataracts - Integumentary: Dry, warm, inelastic skin. Pigmented lesions (on legs), ulcers (especially on feet), loss of hair on toes, acanthosis nigricans - Respiratory: Rapid, deep respirations (Kussmaul respirations)* - Cardiovascular: Hypotension.* Weak, rapid pulse* - Gastrointestinal: Dry mouth, vomiting.* Fruity breath* - Neurologic: Altered reflexes, restlessness, confusion, stupor, coma - Musculoskeletal: Muscle wasting

Diabetes Mellitus - Subjective Data functional health patterns

- Health perception-health management: Positive family history, malaise - Nutrition-metabolic: Obesity, weight loss (type 1), weight gain (type 2). Thirst, hunger, nausea and vomiting. Poor healing (especially involving the feet), eating habits in patients with previously diagnosed diabetes - Elimination: Constipation or diarrhea, frequent urination, frequent bladder infections, nocturia, urinary incontinence - Activity-exercise: Muscle weakness, fatigue - Cognitive-perceptual: Abdominal pain, headache, blurred vision, numbness or tingling of extremities, pruritus - Sexuality-reproductive: Erectile dysfunction, frequent vaginal infections, decreased libido - Coping-stress tolerance: Depression, irritability, apathy - Value-belief: Commitment to lifestyle changes involving diet, medication, and activity patterns

Nutritional Therapy - Total carbohydrate

- Include carbohydrate from fruits, vegetables, grains, legumes, and low-fat milk. - Monitor by carbohydrate counting, exchange lists, or use of appropriate proportions. - Sucrose-containing food can be substituted for other carbohydrates in meal plan. - Fiber intake at 25-30 g/day. - Nonnutritive sweeteners are safe when consumed within FDA daily intake levels.

Nutritional Therapy - Protein

- Individualize goals. - High-protein diets are not recommended for weight loss.

Nutritional Therapy - Fat

- Individualize goals. - Minimize trans fat. - Dietary cholesterol <200 mg/day. - ≥2 servings of fish per week to provide polyunsaturated fatty acids.

Drug therapy

- Insulin - Oral and noninsulin injectable agents - Enteric-coated aspirin - Angiotensin-converting enzyme (ACE) inhibitors - Angiotensin II receptor blockers (ARBs) - Antihyperlipidemic drugs

Nutritional Therapy - Alcohol

- Limit to moderate amount (maximum 1 drink per day for women and 2 drinks per day for men). - Consume alcohol with food to reduce risk of nocturnal hypoglycemia in those using insulin or insulin secretagogues. - Moderate alcohol consumption has no acute effect on glucose and insulin concentrations, but carbohydrate taken with the alcohol (mixed drink) may raise blood glucose.

Diabetes Mellitus - Subjective Data important health information

- Past health history: Mumps, rubella, coxsackievirus, or other viral infections. Recent trauma, infection, or stress. Pregnancy, gave birth to infant >9 lb. Chronic pancreatitis, Cushing syndrome, acromegaly, family history of type 1 or type 2 diabetes mellitus - Medications: Use of and adherence to regimen with insulin or OAs, corticosteroids, diuretics, phenytoin (Dilantin) Surgery or other treatments: Any recent surgery

Type 1 Diabetes Mellitus - Genetic Link

- Predisposition to type 1 diabetes is related to human leukocyte antigens (HLAs) - when an individual with certain HLA types is exposed to a viral infection, the β-cells of the pancreas are destroyed, either directly or through an autoimmune process. The HLA types associated with an increased risk for type 1 diabetes include HLA-DR3 and HLA-DR4

Acanthosis Nigricans

- dark patches of thick velvety skin - neck, armpits, lips, hands, groin, elbows

Major contributing factor

- heart diseases - stroke

Diabetes Mellitus

- is a chronic multisystem disease characterized by hyperglycemia related to abnormal insulin production, impaired insulin utilization, or both. - is a serious health problem throughout the world, and its prevalence is rapidly increasing. - Seventh leading cause of death

α-Glucosidase Inhibitors

- known as "starch blockers," these drugs work by slowing down the absorption of carbohydrate in the small intestine. - Acarbose (Precose) and miglitol (Glyset) are the available drugs in this class. - Taken with the first bite of each main meal, they are most effective in lowering postprandial blood glucose. - Their effectiveness is measured by checking 2-hour postprandial glucose levels.

Meglitinides

- repaglinide (Prandin) and nateglinide (Starlix) increase insulin production by the pancreas. - less likely to cause hypoglycemia because of rapid absorption - When they are taken just before meals, pancreatic insulin production increases during and after the meal, mimicking the normal response to eating. Instruct patients to take meglitinides any time from 30 minutes before each meal right up to the time of the meal. These drugs should not be taken if a meal is skipped.

Diabetes exchange lists

- the individual is given a meal plan with specific numbers of helpings from a list of exchanges for each meal and snack. - The exchanges are starches, fruits, milk, meat, vegetables, fats, and free foods.

Current theories link the causes of diabetes...

...singly or in combination, to genetic, autoimmune, and environmental factors (e.g., virus, obesity).

Identify five hormonees that are replaced when panhypopituitarism from radiation therapy or total hypophysectomy as treatment for pituitary tumors

1) cortisol 2) thyroid 3) vasopressin/ ADH analog 4) Growth hormone 5) sex hormones: testosterone, follicle stimulating hormone (FSH), and leutinizing hormone (LH) if fertility is desired

Average amount of insulin secreted daily by an adult

40 to 50 U, or 0.6 U/kg of body weight.

What is a cause of primary hypothyroidism in adults? a. Malignant or benign thyroid nodules b. Surgical removal or failure of the pituitary gland c. Surgical removal or radiation of the thyroid gland d. Autoimmune-induced atrophy of the thyroid gland

d. Both Graves' disease and Hashimoto's thyroiditis are autoimmune disorders that eventually destroy the thyroid gland, leading to primary hypothyroidism. Thyroid tumors most often result in hyperthyroidism. Secondary hypothyroidism occurs as a result of pituitary failure and iatrogenic hypothyroidism results from thyroidectomy or radiation of the thyroid gland.

Stable normal glucose range

70 to 110 mg/dL

90 The patient has hematocrit 40% and red blood cell count 4.4 x 106 cells/μL. The formula to calculate mean corpuscular volume = Hct x 10/ RBC x 106 = 40 x 10/4.4 x 106 x 106 = 90 fL. Text Reference - p. 628

A 20-year-old male patient has hematocrit of 40 percent and a red blood cell count 4.4 x 106 cells/μL. What is the mean corpuscular volume of the patient? Record your answer using a whole number. ______ fL

2, 5 Splenectomy can result in increased platelet levels and impaired immunologic function as a consequence of the loss of storage and immunologic functions of the spleen. Fibrinolysis, fatigue, and cold intolerance are less likely to result from the loss of the spleen because coagulation and oxygenation are not primary responsibilities of the spleen. Text Reference - p. 618

A 30-year-old patient has undergone a splenectomy as a result of injuries suffered in a motor vehicle accident. Which phenomena are likely to result from the absence of the patient's spleen? Select all that apply. 1 Impaired fibrinolysis 2 Increased platelet levels 3 Increased eosinophil levels 4 Fatigue and cold intolerance 5 Impaired immunologic function

3 Hemoglobin levels commonly decrease in both men and women after middle age. However, the nurse should also check for signs of gastrointestinal bleeding. Total serum iron and total iron binding capacity are decreased in older adults but do not account for potential alterations due to disease process and do not result in anemia. Stomatitis is an inflammation of the mucous lining of any of the structures in the mouth, which may involve the cheeks, gums, tongue, lips, throat, and roof or floor of the mouth. Severe iron deficiency anemia can lead to stomatitis, but stomatitis does not cause anemia. Text Reference - p. 619

A 65-year-old adult with anemia experiences hypoxemia and poor intestinal iron absorption. The nurse recognizes that a low hemoglobin level that is common in the older population may be contributing to the patient's condition. Which other contributing factor should the nurse assess in the patient? 1 Decreased total serum iron 2 Stomatitis 3 Gastrointestinal bleeding 4 Decreased total iron binding capacity

4 Erythropoietin is a hormone secreted by the kidneys that stimulates the bone marrow cells to produce red blood cells (RBCs). Chronic kidney disease causes a reduction in erythropoietin production, which ultimately leads to a decreased production of RBCs. This results in anemia. Excessive blood loss results in anemia, but is not a likely cause in this patient. A decreased iron intake causes anemia, but is less likely in this patient. Hypothyroidism can indirectly lead to anemia, but it is not a cause in this patient, because the patient does not suffer from thyroid abnormality. The cause of anemia in this patient is related to the kidney disease. Text Reference - p. 614

A nurse is assessing a patient with chronic kidney disease. The nurse finds that the patient has pallor, and the blood reports are suggestive of anemia. What could be the most likely cause of anemia? 1 Excess blood loss 2 Decreased intake of iron 3 Decreased thyroid gland activity 4 Decreased erythropoietin production

2 A hard and fixed lymph node is an abnormal finding and warrants further investigation. Hard or fixed nodes suggest malignancy. Palpated nodes which are mobile, firm, and nontender are considered normal. Tender nodes usually indicate inflammation. Vitamin deficiency does not affect lymph nodes. Text Reference - p. 620

A nurse is performing a physical assessment of a patient and finds that the lymph nodes are hard and fixed. How should the nurse interpret the finding? 1 It is a normal finding. 2 It suggests malignancy. 3 It indicates inflammation. 4 It signifies vitamin deficiency.

3, 5, 2, 1, 4 A bone marrow aspiration involves obtaining bone marrow for cytological and chromosomal investigations. The bone marrow is usually aspirated from the posterior iliac crest. During a bone marrow aspiration, the skin over the puncture site is prepared by using a bactericidal agent. A local anesthetic agent is then infiltrated into the skin, subcutaneous tissue, and periosteum. Following this, a bone marrow needle is inserted into the bone through the cortex. The stylet of the needle is then removed and the hub is attached to 10-mLsyringe. The bone marrow is then aspirated. A volume of 0.2 to 0.5 mL is sufficient for laboratory investigations. Text Reference - p. 630

A nurse is providing preprocedural teaching to a patient who is scheduled for a bone marrow aspiration. Arrange the steps of performing a bone marrow aspiration in the correct order. 1. The stylet of the needle is then removed. 2. A bone marrow needle is inserted through the cortex of the bone. 3. The skin over the puncture site is cleansed with a bactericidal agent. 4. The hub is attached to a 10-mL syringe, and 0.2 to 0.5 mL of the fluid marrow is aspirated. 5. The skin, subcutaneous tissue, and periosteum are infiltrated with a local anesthetic agent.

3 In acute leukemia, a complete blood count shows a large number of immature blast white blood cells present in the blood. Pancytopenia is associated with a marked decrease in the number of red blood cells, white blood cells, and platelets. Neutropenia is a condition in which the absolute neutrophil count is less than 1000 cells/μL. Thrombocytopenia is a condition in which the platelet counts falls below 100,000/μL. Text Reference - p. 625

A nurse is reviewing a patient's laboratory reports, which show a large number of immature blast white blood cells in the blood. Which condition is likely to be found in the patient? 1 Pancytopenia 2 Neutropenia 3 Acute leukemia 4 Thrombocytopenia

1, 3, 5 Hemolysis means the destruction of RBCs. The most common sites of hemolysis in the body are the liver, spleen, and bone marrow. Kidneys filter the blood to produce urine, and are not involved in hemolysis. The stomach acts to help in digestion, and it is not involved in hemolysis. Text Reference - p. 614

A nurse provides education to a group of nursing students related to specific parts of the body that are involved in the process of hemolysis. Which body parts should be included in the teaching? Select all that apply. 1 Liver 2 Kidney 3 Spleen 4 Stomach 5 Bone marrow

3 Hemoglobin acts as a buffer and plays a role in maintaining acid-base balance. White blood cells help in immune function. Prothrombin is a coagulation factor and helps in clotting. Platelets help in clotting. Text Reference - p. 614

A nurse recalls that which factor is responsible for maintaining acid-base balance in the body? 1 White blood cells 2 Prothrombin 3 Hemoglobin 4 Platelets

1 A patient's medical report shows the white blood cell count to be 15 × 109/L. The normal range for a white blood cell count is between 4 × 109/L to 11 × 109/L. Elevations in white blood cell count are associated with infection, because white blood cells (WBCs) are immune cells. Neutropenia is a condition in which the absolute neutrophil count (ANC) is less than 1000 cells/μL. It does not increase the risk of bleeding. Thrombocytopenia is a condition in which platelet counts fall below 100,000/μL.

A nurse reviews the lab report of a patient that shows the white blood cell count is 15 × 109/L. Which condition is the patient likely experiencing? 1 Infection 2 Neutropenia 3 Risk of bleeding 4 Thrombocytopenia

4 Hemoglobin saturation results in hypoxemia and compromises brain function, causing the patient to experience difficulty with intracranial regulation. Red blood cell indices are special indicators that reflect red blood cell volume, color, and hemoglobin saturation. Hematocrit value gives the measure of packed cell volume of red blood cells. Hematocrit value is generally three times the hemoglobin value. Red blood cell morphology provides information regarding the size and shape of the patient's red blood cells. Total red blood cell count gives the number of circulating red blood cells. Test-Taking Tip: Identify option components as correct or incorrect. This may help you identify a wrong answer. Text Reference - p. 625

A patient has impaired intracranial regulation and hypoxemia. The primary health care provider prescribes red blood cell indices for the patient, and the patient later asks the nurse to explain the purpose of the test. What should the nurse tell the patient? 1 To measure the packed cell volume in the blood 2 To test the size and shape of the red blood cells 3 To measure the number of circulating red blood cells 4 To measure the gas-carrying capacity of the red blood cells

however, they do not interfere with the bone scan. Text Reference - p. 629

A patient is scheduled for a bone scan. A nurse explains the procedure to the patient. Which statements made by the patient indicate effective learning? Select all that apply. 1 "I will have to lie down in a small chamber during the test." 2 "I will have to maintain a lying down position during the procedure." 3 "I should remove all metal objects before the procedure." 4 "I should not have surgical staples inside my body as they can interfere with the procedure." 5 "I should drink four to six glasses of water and then void before the imaging."

therefore, a platelet count of 100,000/μL in the laboratory reports indicates a low count. A hemoglobin level of 18 g/dL and a RBC count of 6 million/μL indicate polycythemia, which causes flushing of the skin and visual abnormalities. A WBC count of 3500/μL indicates leukopenia, which is associated with bone marrow depression, severe or chronic illness, and some types of leukemia. Test-Taking Tip: Bleeding occurs due to abnormal clotting. Recollect the clotting process and components of blood involved in coagulation process to choose the correct option. Text Reference - p. 623

A patient reports bleeding from the nostrils. Upon further interaction, the patient reports that bleeding was spontaneous and started after lifting a heavy box from the ground. Which finding would the nurse expect in the laboratory reports of this patient? 1 A hemoglobin level of 18 g/dL 2 A platelet count of 100,000/μL 3 A white blood cell (WBC) count of 3500/μL 4 A red blood cell count (RBC) of 6 million cells/μL

3 Small vessel occlusions causing a purple, mottled appearance of the face, nose, fingers, or toes are the symptoms of erythrocytosis. It is a condition in which the proportion of blood volume occupied by red blood cells increases. Leukopenia is a condition in which the white blood cells count less than 4000/μL. Neutropenia is a condition in which the absolute neutrophil count (ANC) is less than 1000 cells/μL. Thrombocytopenia is a condition in which platelet counts falls below 100,000/μL. Text Reference - p. 623

A patient reports small vessel occlusions causing a purple, mottled appearance of the face, nose, and fingers. The nurse suspects that the patient is experiencing what disorder? 1 Leukopenia 2 Neutropenia 3 Erythrocytosis 4 Thrombocytopenia

4 After bone marrow aspiration, if bleeding is present at the site, the patient should be advised to lie on the affected side for 30 to 60 minutes to maintain pressure on the site. If the bed is too soft, the patient can lie on a rolled towel to provide additional pressure. Walking, sitting, and standing do not help to maintain pressure on the site. Text Reference - p. 630

A patient who has undergone bone marrow aspiration is being monitored by the nurse. The nurse observes that bleeding is present at the needle aspiration site. Which action should the nurse advise the patient to perform? 1 Walking for 10 to 15 minutes 2 Sitting for 30 to 40 minutes 3 Standing for 30 to 40 minutes 4 Lying on the side for 30 to 60 minutes

1 The spleen has four major functions: storage, filtration, immunologic, and hematopoietic. The spleen acts as a storage site for red blood cells and platelets. Thus, a patient with a splenectomy has higher circulating levels of platelets due to impaired storage function. If there is a high level of old and defective red blood cells, then the filtration function is affected. The spleen's immunologic function is demonstrated by a rich supply of lymphocytes, monocytes, and stored immunoglobulins. The spleen's hematopoietic function is demonstrated by a failure to produce red blood cells during fetal development. Text Reference - p. 618

A patient who underwent splenectomy has a platelet count of 500,000/μL. The nurse recognizes that due to the absence of the spleen, the increased number is due to the impairment of which major function of spleen? 1 Storage 2 Filtration 3 Immunologic 4 Hematopoietic

2, 4 In PET scan, a nuclear tracer substance is injected and is taken up by metabolically active cells. Before the test, the nurse should ensure that IV access is available, because it is required for injection of the tracer substance. Patients should have nothing by mouth except water and medications for at least four hours before the test, and IV solutions containing glucose can be held. Avoiding water, stopping medications, and feeding the patient before the test are not recommended. Text Reference - p. 629

A patient will have a positron emission tomography (PET) scan for diagnostic studies. Which actions should the nurse perform prior to the scan? Select all that apply. 1 Give IV fluids containing glucose. 2 Ensure IV access is available for injection. 3 Avoid water and stop medications. 4 Stop oral intake of food four hours before the test. 5 Administer the test directly after a meal.

1, 4 While checking for a history of chemical exposure, the nurse asks whether the patient has worked in the military or not. Many Vietnam War veterans were exposed to a toxin-containing defoliant. This toxin increases the risk of leukemia and lymphoma. Therefore, the nurse should ask if the patient has been in the military. A patient's occupation may cause exposure to chemicals like benzene, lead, naphthalene, and phenylbutazone. These chemicals are commonly used by potters, dry cleaners, and individuals involved with occupations that use adhesives. Some illnesses may cause a change in appetite and affect eating habits. Information about appetite and diet helps to determine dietary habits, but does not help in determining chemical exposure history. The dietary pattern of the patient is good, so there is no need to take a dietary history. Text Reference - p. 621

A patient with a history of a well-balanced nutritional intake is diagnosed with anemia. To determine potential exposure to chemicals, which questions should the nurse include in the history-taking? Select all that apply. 1 "Have you worked in the military?" 2 "Is your appetite affected?" 3 "What kind of diet do you follow?" 4 "Does your occupation expose you to hazardous substances?" 5 "Has your present illness caused a change in your eating habits?"

2 If a patient is tachycardic, the heart rate is above 100 beats/minute, which may occur in anemic patients as a compensatory mechanism to increase cardiac output. If a patient is bradycardic, the heart rate is below 60 beats/minute. Hypertensive and hypotensive refer to blood pressure readings, not the heart rate. STUDY TIP: Laughter is a great stress reliever. Watching a short program that makes you laugh, reading something funny, or sharing humor with friends helps decrease stress. Text Reference - p. 624

A patient with anemia presents with a heart rate of 120 beats/minute. As what should the nurse document the heart rate? 1 Bradycardic 2 Tachycardic 3 Hypertensive 4 Hypotensive

1 Positron emission tomography is an imaging technique that traces the malignancy based on increased glucose consumption. A patient with diabetes will have altered glucose levels, which will alter the test results. Therefore, the nurse reschedules the patient's medication five to six hours before the test. A patient with hypertension will not have any alteration in metabolism. Therefore, the patient can drink water and medication before the test. Peptic ulcer disease may result in bright red vomitus. However, it does not cause any alteration of metabolism. Rheumatoid arthritis is an autoimmune disorder, which results in anemia but does not alter metabolism. Test-Taking Tip: PET is an imaging technique useful to diagnose cancer cells. Recall the working of this technique, how can you distinguish cancer cells and tumor cells. This would help you to correlate with the condition. Text Reference - p. 620

A patient with leukemia is scheduled for a positron emission tomography. The nurse makes changes in the medication schedule so that the patient receives medication five to six hours before the test. What could be the reason behind this intervention? 1 The patient has diabetes. 2 The patient has hypertension. 3 The patient has peptic ulcer disease. 4 The patient has rheumatoid arthritis.

3 The patient's complete blood count is suppressed. There is a marked decrease in the number of RBCs, WBCs, and platelets. This condition is called pancytopenia. Leukopenia is a condition in which white blood cells count less than 4000/μL. Neutropenia is a condition in which the absolute neutrophil count (ANC) is less than 1000 cells/μL. Thrombocytopenia is condition in which platelet counts falls below 100,000/μL. Text Reference - p. 625

A patient's complete blood count is RBC 1.8 × 106/μL, WBC 2 × 109/L, platelets 90 × 109/L. How should the nurse interpret the test results? 1 Leukopenia 2 Neutropenia 3 Pancytopenia 4 Thrombocytopenia

A patient with acromegaly is treated with transsphenoidal hypophysectomy. Postoperatively the nurse a. ensures that any clear nasal drainage is tested for glucose b. maintains the patient flat in bed to prevent CSF leakage c. assists the patient with toothbrushing every 4 hours to keep the surgical area clean d. encourages deep breathing, coughing, and turning to prevent respiratory complications

A- A transsphenoidal hypophysectomy involves entry into the sella turcica through an incision in the upper lip and gingiva into the floor of the nose and the sphenoid sinuses. Postoperative clear nasal drainage with glucose content indicates CSF leakage from an open connection to the brain, putting the patient at risk for meningitis. After surgery, the patient is positioned with the head of the bed elevated to avoid pressure on the sella turcica. Coughing and straining are avoided to prevent increased ICP and CSF leakage, and although mouth care is required every 4 hours, toothbrushing should not be performed because injury to the suture line may occur.

Preoperative instructions for the patient scheduled for a subtotal thyroidectomy include teaching the patient a. how to support the head with the hands when moving b. that coughing should be avoided to prevent pressure on the incision c. that the head and neck will need to remain immobile until the incision heals d. that any tingling around the lips or in the fingers after surgery is expected and temporary

A- To prevent strain on the suture line postoperatively, the patient's head must be manually supported while turning and moving in bed, but ROM exercises for the head and neck are also taught preoperatively to be gradually implemented after surgery. There is no contraindication for coughing and deep breathing, and these should be carried out postoperatively. Tingling around the lips or fingers is a sign of hypocalcemia, which may occur if the parathyroid glands are inadvertently removed during surgery. This sign should be reported immediately.

During the care of the patient with SIADH, the nurse should a. monitor neurologic status at least every 2 hours b. keep the head of the bed elevated to prevent ADH release c. teach the patient receiving treatment with diuretics to restrict sodium intake d. notify the health care provider if the patient's BP decreases more than 20 mmHg from baseline

A- the patient with syndrome of inappropriate secretion of antidiuretic hormone (SIADH) has marked dilutional hyponatremia and should be monitored for decreased neurologic function and convulsions every 2 hours. ADH release is reduced by keeping the head of the bed flat to increase left atrial filling pressure, and sodium intake is supplemented because of the hyponatremia and sodium loss caused by diuretics. A reduction in BP indicates a reduction in total fluid volume and is an expected outcome of treatment.

The nurse assesses a 76-year-old man with chronic myeloid leukemia receiving nilotinib (Tasigna). It is most important for the nurse to ask which question? A. "Have you had a fever?" B. "Have you lost any weight?" C. "Has diarrhea been a problem?" D. "Have you noticed any hair loss?"

A. "Have you had a fever?" An adverse effect of nilotinib is neutropenia. Infection is common in neutropenic patients and is the primary cause of death in cancer patients. Patients should report a temperature of 100.4o F or higher. Other adverse effects of nilotinib are thrombocytopenia, bleeding, nausea, fatigue, elevated lipase level, fever, rash, pruritus, diarrhea, and pneumonia.

Which individual is at high risk for a cobalamin (vitamin B12) deficiency anemia? A. A 47-year-old man who had a gastrectomy (removal of the stomach) B. A 54-year-old man with a history of irritable bowel disease and ulcerative colitis C. A 26-year-old woman who complains of heavy menstrual periods D. A 15-year-old girl who is a vegetarian

A. A 47-year-old man who had a gastrectomy (removal of the stomach) There are many causes of cobalamin deficiency. The most common cause is pernicious anemia, a disease in which the gastric mucosa is not secreting intrinsic factor (IF) because of antibodies being directed against the gastric parietal cells or IF itself. Other causes of cobalamin deficiency include gastrectomy, gastritis, nutritional deficiency, chronic alcoholism, and hereditary enzymatic defects of cobalamin use. Reference: 668

Previous administrations of chemotherapy agents to a cancer patient have resulted in diarrhea. Which dietary modification should the nurse recommend? A. A bland, low-fiber diet B. A high-protein, high-calorie diet C. A diet high in fresh fruits and vegetables D. A diet emphasizing whole and organic foods

A. A bland, low-fiber diet Patients experiencing diarrhea secondary to chemotherapy and/or radiation therapy often benefit from a diet low in seasonings and roughage before the treatment. Foods should be easy to digest and low in fat. Fresh fruits and vegetables are high in fiber and should be minimized during treatment. Whole and organic foods do not prevent diarrhea.

Which patient is most likely to experience anemia caused by increased destruction of RBCs? A. An African American man who has a diagnosis of sickle cell disease B. A 59-year-old man whose alcoholism has precipitated folic acid deficiency C. A 30-year-old woman with a history of "heavy periods" accompanied by anemia D. A 3-year-old child whose impaired growth and development is attributable to thalassemia

A. An African American man who has a diagnosis of sickle cell disease The cause of sickle cell anemia involves increased hemolysis. Thalassemias and folic acid deficiencies decrease erythropoiesis, whereas the anemia related to menstruation is a direct result of blood loss. Reference: 672

Which points should be included in teaching the patient with sickle cell disease (select all that apply)? A. Avoid dehydration. B. Avoid high altitudes. C. Take cobalamin (vitamin B12) regularly. D. Consume dairy products frequently. E. Increase consumption of grapefruit juice.

A. Avoid dehydration. B. Avoid high altitudes. Avoiding dehydration and high altitudes helps to prevent crises. Vitamins, dairy products, and grapefruit juice cannot help the patient to prevent attacks of sickle cell disease. Reference: 675

The laboratory reports that the cells from the patient's tumor biopsy are Grade II. What should the nurse know about this histologic grading? A. Cells are abnormal and moderately differentiated. B. Cells are very abnormal and poorly differentiated. C. Cells are immature, primitive, and undifferentiated. D. Cells differ slightly from normal cells and are well-differentiated.

A. Cells are abnormal and moderately differentiated. Grade II cells are more abnormal than Grade I and moderately differentiated. Grade I cells differ slightly from normal cells and are well-differentiated. Grade III cells are very abnormal and poorly differentiated. Grade IV cells are immature, primitive, and undifferentiated

The patient and his family are upset that the patient is going through procedures to diagnose cancer. What nursing actions should the nurse use first to facilitate their coping with this situation (select all that apply)? A. Maintain hope. B. Exhibit a caring attitude. C. Plan realistic long-term goals. D. Give them antianxiety medications. E. Be available to listen to fears and concerns. F. Teach them about all the types of cancer that could be diagnosed.

A. Maintain hope. B. Exhibit a caring attitude. E. Be available to listen to fears and concerns. Maintaining hope, exhibiting a caring attitude, and being available to actively listen to fears and concerns would be the first nursing interventions to use as well as assessing factors affecting coping during the diagnostic period. Providing relief from distressing symptoms for the patient and teaching them about the diagnostic procedures would also be important. Realistic long-term goals and teaching about the type of cancer cannot be done until the cancer is diagnosed. Giving the family antianxiety medications would not be appropriate.

Which assessment findings are consistent with thrombocytopenia? A. Petechiae, ecchymoses B. Pallor, spider angiomas C. Cyanosis, dullness D. Jaundice, purpura

A. Petechiae, ecchymoses Many patients with thrombocytopenia are asymptomatic. The most common symptom is bleeding, usually mucosal or cutaneous. Mucosal bleeding may manifest as epistaxis and gingival bleeding, and large bullous hemorrhages may appear on the buccal mucosa because of the lack of vessel protection by the submucosal tissue. Bleeding into the skin manifests as petechiae, purpura, or superficial ecchymoses Reference: 679

Which foods should you encourage patients with folic acid deficiency to include in their daily food intake (select all that apply)? A. Ready-to-eat cereal B. Wheat tortillas C. Lentils D. Strawberries E. Potatoes

A. Ready-to-eat cereal B. Wheat tortillas C. Lentils Whole-grain foods and beans are high in folic acid. Reference: 666, 670

You are evaluating the laboratory data of the patient with suspected aplastic anemia. Which findings support this diagnosis? A. Reduced RBCs, reduced white blood cells (WBCs), and reduced platelets B. Reduced RBCs, normal WBCs, and normal platelets C. Normal RBCs, reduced WBCs, and reduced platelets D. Elevated RBCs, increased WBCs, and increased platelets

A. Reduced RBCs, reduced white blood cells (WBCs), and reduced platelets Because all marrow elements are affected, hemoglobin, WBC, and platelet values are decreased in aplastic anemia. Other RBC indices usually are normal. Reference: 671

A patient is admitted to the hospital with a diagnosis of Cushing syndrome. On physical assessment of the patient, the nurse would expect to find a. hypertension, peripheral edema, and petechiae b. weight loss, buffalo hump, and moon face with acne c. abdominal and buttock striae, truncal obesity, and hypotension d. anorexia, signs of dehydration, and hyperpigmentation of the skin

A. The effects of glucocorticoid excess include weight gain from accumulation and redistribution of adipose tissue, sodium and water retention, glucose intolerance, protein wasting, loss of bone structure, loss of collagen, and capillary fragility. Clinical manifestations of corticosteroid deficiency include hypotension, dehydration, weight loss, and hyperpigmentation of the skin.

The Schilling test for pernicious anemia involves A. administration of radioactive cobalamin and measuring its excretion in the urine. B. blood cultures for organism identification. C. the measurement of serum iron. D. the administration of iron and blood assessment of total iron binding in 24 hours.

A. administration of radioactive cobalamin and measuring its excretion in the urine. Parietal cell function can be assssed with a Schilling test. After radioactive cobalamin is administered to the patient, the amount of cobalamin excreted in the urine is measured. An individual who cannot absorb cobalamin excretes only a small amount of this radioactive form. Reference: 669

When providing care for a patient with thrombocytopenia, you instruct the patient to A. dab his or her nose instead of blowing. B. be careful when shaving with a safety razor. C. continue with physical activities to stimulate thrombopoiesis. D. avoid aspirin because it may mask the fever that occurs with thrombocytopenia.

A. dab his or her nose instead of blowing. Blowing the nose forcefully should be avoided

You encourage the patient with cobalamin deficiency to seek treatment because untreated pernicious anemia may result in A. death. B. liver failure. C. heart failure. D. gastrectomy.

A. death. Regardless of how much cobalamin is ingested, the patient is not able to absorb it if intrinsic factor is lacking or if there is impaired absorption in the ileum. For this reason, increasing dietary cobalamin does not correct the anemia. However, the patient should be instructed about adequate dietary intake to maintain good nutrition (see Table 31-5). Parenteral (cyanocobalamin or hydroxocobalamin) or intranasal (Nascobal, CaloMist) administration of cobalamin is the treatment of choice. Without cobalamin administration, these individuals will die in 1 to 3 years. Reference: 669

In a severely anemic patient, you expect to find A. dyspnea and tachycardia. B. cyanosis and pulmonary edema. C. cardiomegaly and pulmonary fibrosis. D. ventricular dysrhythmias and wheezing.

A. dyspnea and tachycardia. Patients with severe anemia (hemoglobin <6 g/dL) exhibit the following cardiovascular and pulmonary manifestations: tachycardia, increased pulse pressure, systolic murmurs, intermittent claudication, angina, heart failure, myocardial infarction

The nursing management of a patient in sickle cell crisis includes (select all that apply) A. monitoring of the complete blood cell (CBC) count. B. blood transfusions if required and iron chelation. C. optimal pain management and oxygen therapy. D. rest as needed and deep vein thrombosis prophylaxis.

A. monitoring of the complete blood cell (CBC) count. B. blood transfusions if required and iron chelation. C. optimal pain management and oxygen therapy. D. rest as needed and deep vein thrombosis prophylaxis. (all of the above) The CBC count is monitored. Infections are common with an elevated white blood cell (WBC) count, and anemia may occur with low hemoglobin and red blood cell (RBC) levels. Oxygen may be administered to treat hypoxia and control sickling. Rest may be instituted to reduce metabolic requirements and deep vein thrombosis prophylaxis (using anticoagulants) prescribed. Transfusion therapy is indicated when an aplastic crisis occurs. Patients may require iron chelation therapy to reduce transfusion-produced iron overload. Pain occurring during an acute crisis usually is undertreated. Patients should have optimal pain control with opioid analgesics, nonsteroidal antiinflammatory agents, antineuropathic pain medications, local anesthetics, or nerve blocks. Reference: 674-675

Nursing interventions for a patient with severe anemia related to peptic ulcer disease include (select all that apply) A. monitoring stools for guaiac. B. instructions about a high-iron diet. C. taking vital signs every 8 hours. D. teaching self-injection of erythropoietin.

A. monitoring stools for guaiac. B. instructions about a high-iron diet. The stool guaiac test is done to determine whetherthe cause of the iron-deficiency anemia is related to gastrointestinal bleeding. Iron should be increased in the diet. Teach the patient which foods are good sources of iron. If nutrition is already adequate, increasing iron intake by dietary means may not be practical. The patient with iron deficiency related to acute blood loss may require a transfusion of packed red blood cells (RBCs). Reference: 666

You correctly identify which descriptions as characteristic of anemia of chronic disease (select all that apply) A. normocytic. B. normochromic. C. microcytic. D. hypochromic. E. proliferative.

A. normocytic. B. normochromic. Anemia of chronic disease, also called anemia of inflammation, is associated with an underproduction of RBCs and mild shortening of RBC survival. The RBCs are usually normocytic, normochromic, and hypoproliferative. The anemia is usually mild, but it can be more severe. Reference: 670

A complication of the hyperviscosity of polycythemia is A. thrombosis. B. cardiomyopathy. C. pulmonary edema. D. disseminated intravascular coagulation (DIC).

A. thrombosis. Thrombosis is the most likely complication. The patient with polycythemia may experience angina, heart failure, intermittent claudication, and thrombophlebitis, which may be complicated by embolization. The most common and serious acute complication is stroke due to thrombosis. Reference: 677

Which prescribed medication should the nurse administer first to a 60-year-old patient admitted to the emergency department in thyroid storm? a. Propranolol (Inderal) b. Propylthiouracil (PTU) c. Methimazole (Tapazole) d. Iodine (Lugol's solution)

ANS: A Adrenergic blockers work rapidly to decrease the cardiovascular manifestations of thyroid storm. The other medications take days to weeks to have an impact on thyroid function.

Which finding for a patient who has hypothyroidism and hypertension indicates that the nurse should contact the health care provider before administering levothyroxine (Synthroid)? a. Increased thyroxine (T4) level b. Blood pressure 112/62 mm Hg c. Distant and difficult to hear heart sounds d. Elevated thyroid stimulating hormone level

ANS: A An increased thyroxine level indicates the levothyroxine dose needs to be decreased. The other data are consistent with hypothyroidism and the nurse should administer the levothyroxine.

25. Which question during the assessment of a diabetic patient will help the nurse identify autonomic neuropathy? a. "Do you feel bloated after eating?" b. "Have you seen any skin changes?" c. "Do you need to increase your insulin dosage when you are stressed?" d. "Have you noticed any painful new ulcerations or sores on your feet?"

ANS: A Autonomic neuropathy can cause delayed gastric emptying, which results in a bloated feeling for the patient. The other questions are also appropriate to ask but would not help in identifying autonomic neuropathy.

9. In order to assist an older diabetic patient to engage in moderate daily exercise, which action is most important for the nurse to take? a. Determine what type of activities the patient enjoys. b. Remind the patient that exercise will improve self-esteem. c. Teach the patient about the effects of exercise on glucose level. d. Give the patient a list of activities that are moderate in intensity.

ANS: A Because consistency with exercise is important, assessment for the types of exercise that the patient finds enjoyable is the most important action by the nurse in ensuring adherence to an exercise program. The other actions will also be implemented but are not the most important in improving compliance.

Which finding by the nurse when assessing a patient with Hashimoto's thyroiditis and a goiter will require the most immediate action? a. New-onset changes in the patient's voice b. Apical pulse rate at rest 112 beats/minute c. Elevation in the patient's T3 and T4 levels d. Bruit audible bilaterally over the thyroid gland

ANS: A Changes in the patient's voice indicate that the goiter is compressing the laryngeal nerve and may lead to airway compression. The other findings will also be reported but are expected with Hashimoto's thyroiditis and do not require immediate action.

45. Which nursing action can the nurse delegate to unlicensed assistive personnel (UAP) who are working in the diabetic clinic? a. Measure the ankle-brachial index. b. Check for changes in skin pigmentation. c. Assess for unilateral or bilateral foot drop. d. Ask the patient about symptoms of depression.

ANS: A Checking systolic pressure at the ankle and brachial areas and calculating the ankle-brachial index is a procedure that can be done by UAP who have been trained in the procedure. The other assessments require more education and critical thinking and should be done by the registered nurse (RN).

Which finding indicates to the nurse that the current therapies are effective for a patient with acute adrenal insufficiency? a. Increasing serum sodium levels b. Decreasing blood glucose levels c. Decreasing serum chloride levels d. Increasing serum potassium levels

ANS: A Clinical manifestations of Addison's disease include hyponatremia and an increase in sodium level indicates improvement. The other values indicate that treatment has not been effective.

A 56-year-old female patient has an adrenocortical adenoma, causing hyperaldosteronism. The nurse providing care should a. monitor the blood pressure every 4 hours. b. elevate the patient's legs to relieve edema. c. monitor blood glucose level every 4 hours. d. order the patient a potassium-restricted diet.

ANS: A Hypertension caused by sodium retention is a common complication of hyperaldosteronism. Hyperaldosteronism does not cause an elevation in blood glucose. The patient will be hypokalemic and require potassium supplementation before surgery. Edema does not usually occur with hyperaldosteronism.

34. A 27-year-old patient admitted with diabetic ketoacidosis (DKA) has a serum glucose level of 732 mg/dL and serum potassium level of 3.1 mEq/L. Which action prescribed by the health care provider should the nurse take first? a. Place the patient on a cardiac monitor. b. Administer IV potassium supplements. c. Obtain urine glucose and ketone levels. d. Start an insulin infusion at 0.1 units/kg/hr.

ANS: A Hypokalemia can lead to potentially fatal dysrhythmias such as ventricular tachycardia and ventricular fibrillation, which would be detected with electrocardiogram (ECG) monitoring. Because potassium must be infused over at least 1 hour, the nurse should initiate cardiac monitoring before infusion of potassium. Insulin should not be administered without cardiac monitoring because insulin infusion will further decrease potassium levels. Urine glucose and ketone levels are not urgently needed to manage the patient's care.

13. Which patient action indicates a good understanding of the nurse's teaching about the use of an insulin pump? a. The patient programs the pump for an insulin bolus after eating. b. The patient changes the location of the insertion site every week. c. The patient takes the pump off at bedtime and starts it again each morning. d. The patient plans for a diet that is less flexible when using the insulin pump.

ANS: A In addition to the basal rate of insulin infusion, the patient will adjust the pump to administer a bolus after each meal, with the dosage depending on the oral intake. The insertion site should be changed every 2 or 3 days. There is more flexibility in diet and exercise when an insulin pump is used. The pump will deliver a basal insulin rate 24 hours a day.

37. A female patient is scheduled for an oral glucose tolerance test. Which information from the patient's health history is most important for the nurse to communicate to the health care provider? a. The patient uses oral contraceptives. b. The patient runs several days a week. c. The patient has been pregnant three times. d. The patient has a family history of diabetes.

ANS: A Oral contraceptive use may falsely elevate oral glucose tolerance test (OGTT) values. Exercise and a family history of diabetes both can affect blood glucose but will not lead to misleading information from the OGTT. History of previous pregnancies may provide informational about gestational glucose tolerance, but will not lead to misleading information from the OGTT.

The nurse determines that additional instruction is needed for a 60-year-old patient with chronic syndrome of inappropriate antidiuretic hormone (SIADH) when the patient says which of the following? a. "I need to shop for foods low in sodium and avoid adding salt to food." b. "I should weigh myself daily and report any sudden weight loss or gain." c. "I need to limit my fluid intake to no more than 1 quart of liquids a day." d. "I will eat foods high in potassium because diuretics cause potassium loss."

ANS: A Patients with SIADH are at risk for hyponatremia, and a sodium supplement may be prescribed. The other patient statements are correct and indicate successful teaching has occurred.

14. A 32-year-old patient with diabetes is starting on intensive insulin therapy. Which type of insulin will the nurse discuss using for mealtime coverage? a. Lispro (Humalog) b. Glargine (Lantus) c. Detemir (Levemir) d. NPH (Humulin N)

ANS: A Rapid- or short-acting insulin is used for mealtime coverage for patients receiving intensive insulin therapy. NPH, glargine, or detemir will be used as the basal insulin.

10. Which statement by the patient indicates a need for additional instruction in administering insulin? a. "I need to rotate injection sites among my arms, legs, and abdomen each day." b. "I can buy the 0.5 mL syringes because the line markings will be easier to see." c. "I should draw up the regular insulin first after injecting air into the NPH bottle." d. "I do not need to aspirate the plunger to check for blood before injecting insulin."

ANS: A Rotating sites is no longer recommended because there is more consistent insulin absorption when the same site is used consistently. The other patient statements are accurate and indicate that no additional instruction is needed.

44. The nurse has administered 4 oz of orange juice to an alert patient whose blood glucose was 62 mg/dL. Fifteen minutes later, the blood glucose is 67 mg/dL. Which action should the nurse take next? a. Give the patient 4 to 6 oz more orange juice. b. Administer the PRN glucagon (Glucagon) 1 mg IM. c. Have the patient eat some peanut butter with crackers. d. Notify the health care provider about the hypoglycemia.

ANS: A The "rule of 15" indicates that administration of quickly acting carbohydrates should be done 2 to 3 times for a conscious patient whose glucose remains less than 70 mg/dL before notifying the health care provider. More complex carbohydrates and fats may be used once the glucose has stabilized. Glucagon should be used if the patient's level of consciousness decreases so that oral carbohydrates can no longer be given.

7. A 38-year-old patient who has type 1 diabetes plans to swim laps daily at 1:00 PM. The clinic nurse will plan to teach the patient to a. check glucose level before, during, and after swimming. b. delay eating the noon meal until after the swimming class. c. increase the morning dose of neutral protamine Hagedorn (NPH) insulin. d. time the morning insulin injection so that the peak occurs while swimming.

ANS: A The change in exercise will affect blood glucose, and the patient will need to monitor glucose carefully to determine the need for changes in diet and insulin administration. Because exercise tends to decrease blood glucose, patients are advised to eat before exercising. Increasing the morning NPH or timing the insulin to peak during exercise may lead to hypoglycemia, especially with the increased exercise.

6. A 55-year-old female patient with type 2 diabetes has a nursing diagnosis of imbalanced nutrition: more than body requirements. Which goal is most important for this patient? a. The patient will reach a glycosylated hemoglobin level of less than 7%. b. The patient will follow a diet and exercise plan that results in weight loss. c. The patient will choose a diet that distributes calories throughout the day. d. The patient will state the reasons for eliminating simple sugars in the diet.

ANS: A The complications of diabetes are related to elevated blood glucose, and the most important patient outcome is the reduction of glucose to near-normal levels. The other outcomes also are appropriate but are not as high in priority.

35. A 54-year-old patient is admitted with diabetic ketoacidosis. Which admission order should the nurse implement first? a. Infuse 1 liter of normal saline per hour. b. Give sodium bicarbonate 50 mEq IV push. c. Administer regular insulin 10 U by IV push. d. Start a regular insulin infusion at 0.1 units/kg/hr.

ANS: A The most urgent patient problem is the hypovolemia associated with diabetic ketoacidosis (DKA), and the priority is to infuse IV fluids. The other actions can be done after the infusion of normal saline is initiated.

26. Which information will the nurse include in teaching a female patient who has peripheral arterial disease, type 2 diabetes, and sensory neuropathy of the feet and legs? a. Choose flat-soled leather shoes. b. Set heating pads on a low temperature. c. Use callus remover for corns or calluses. d. Soak feet in warm water for an hour each day.

ANS: A The patient is taught to avoid high heels and that leather shoes are preferred. The feet should be washed, but not soaked, in warm water daily. Heating pad use should be avoided. Commercial callus and corn removers should be avoided. The patient should see a specialist to treat these problems

The nurse is caring for a patient admitted with diabetes insipidus (DI). Which information is most important to report to the health care provider? a. The patient is confused and lethargic. b. The patient reports a recent head injury. c. The patient has a urine output of 400 mL/hr. d. The patient's urine specific gravity is 1.003.

ANS: A The patient's confusion and lethargy may indicate hypernatremia and should be addressed quickly. In addition, patients with DI compensate for fluid losses by drinking copious amounts of fluids, but a patient who is lethargic will be unable to drink enough fluids and will become hypovolemic. A high urine output, low urine specific gravity, and history of a recent head injury are consistent with diabetes insipidus, but they do not require immediate nursing action to avoid life-threatening complications.

12. A patient receives aspart (NovoLog) insulin at 8:00 AM. Which time will it be most important for the nurse to monitor for symptoms of hypoglycemia? a. 10:00 AM b. 12:00 AM c. 2:00 PM d. 4:00 PM

ANS: A The rapid-acting insulins peak in 1 to 3 hours. The patient is not at a high risk for hypoglycemia at the other listed times, although hypoglycemia may occur.

A patient who was admitted with myxedema coma and diagnosed with hypothyroidism is improving and expected to be discharged in 2 days. Which teaching strategy will be best for the nurse to use? a. Provide written reminders of self-care information. b. Offer multiple options for management of therapies. c. Ensure privacy for teaching by asking visitors to leave. d. Delay teaching until patient discharge date is confirmed.

ANS: A Written instructions will be helpful to the patient because initially the hypothyroid patient may be unable to remember to take medications and other aspects of self-care. Because the treatment regimen is somewhat complex, teaching should be initiated well before discharge. Family members or friends should be included in teaching because the hypothyroid patient is likely to forget some aspects of the treatment plan. A simpler regimen will be easier to understand until the patient is euthyroid.

A patient with diabetes insipidus is treated with nasal desmopressin acetate (DDAVP). The nurse determines that the drug is not having an adequate therapeutic effect when the patient experiences a. headache and weight gain. c. a urine specific gravity of 1.002. b. nasal irritation and nausea. d. an oral intake greater than urinary output.

c. Normal urine specific gravity is 1.005 to 1.025 and urine with a specific gravity of 1.002 is very dilute, indicating that there continues to be excessive loss of water and that treatment of diabetes insipidus is inadequate. Headache, weight gain, and oral intake greater than urinary output are signs of volume excess that occur with overmedication. Nasal irritation and nausea may also indicate overdosage.

A 45-year-old male patient with suspected acromegaly is seen at the clinic. To assist in making the diagnosis, which question should the nurse ask? a. "Have you had a recent head injury?" b. "Do you have to wear larger shoes now?" c. "Is there a family history of acromegaly?" d. "Are you experiencing tremors or anxiety?"

ANS: B Acromegaly causes an enlargement of the hands and feet. Head injury and family history are not risk factors for acromegaly. Tremors and anxiety are not clinical manifestations of acromegaly.

The nurse is planning postoperative care for a patient who is being admitted to the surgical unit form the recovery room after transsphenoidal resection of a pituitary tumor. Which nursing action should be included? a. Palpate extremities for edema. b. Measure urine volume every hour. c. Check hematocrit every 2 hours for 8 hours. d. Monitor continuous pulse oximetry for 24 hours.

ANS: B After pituitary surgery, the patient is at risk for diabetes insipidus caused by cerebral edema. Monitoring of urine output and urine specific gravity is essential. Hemorrhage is not a common problem. There is no need to check the hematocrit hourly. The patient is at risk for dehydration, not volume overload. The patient is not at high risk for problems with oxygenation, and continuous pulse oximetry is not needed.

41. A 34-year-old has a new diagnosis of type 2 diabetes. The nurse will discuss the need to schedule a dilated eye exam a. every 2 years. b. as soon as possible. c. when the patient is 39 years old. d. within the first year after diagnosis.

ANS: B Because many patients have some diabetic retinopathy when they are first diagnosed with type 2 diabetes, a dilated eye exam is recommended at the time of diagnosis and annually thereafter. Patients with type 1 diabetes should have dilated eye exams starting 5 years after they are diagnosed and then annually.

47. After change-of-shift report, which patient should the nurse assess first? a. 19-year-old with type 1 diabetes who has a hemoglobin A1C of 12% b. 23-year-old with type 1 diabetes who has a blood glucose of 40 mg/dL c. 40-year-old who is pregnant and whose oral glucose tolerance test is 202 mg/dL d. 50-year-old who uses exenatide (Byetta) and is complaining of acute abdominal pain

ANS: B Because the brain requires glucose to function, untreated hypoglycemia can cause unconsciousness, seizures, and death. The nurse will rapidly assess and treat the patient with low blood glucose. The other patients also have symptoms that require assessments and/or interventions, but they are not at immediate risk for life-threatening complications.

20. The nurse is preparing to teach a 43-year-old man who is newly diagnosed with type 2 diabetes about home management of the disease. Which action should the nurse take first? a. Ask the patient's family to participate in the diabetes education program. b. Assess the patient's perception of what it means to have diabetes mellitus. c. Demonstrate how to check glucose using capillary blood glucose monitoring. d. Discuss the need for the patient to actively participate in diabetes management.

ANS: B Before planning teaching, the nurse should assess the patient's interest in and ability to self-manage the diabetes. After assessing the patient, the other nursing actions may be appropriate, but planning needs to be individualized to each patient.

The nurse is assessing a 41-year-old African American male patient diagnosed with a pituitary tumor causing panhypopituitarism. Assessment findings consistent with panhypopituitarism include a. high blood pressure. b. decreased facial hair. c. elevated blood glucose. d. tachycardia and cardiac palpitations.

ANS: B Changes in male secondary sex characteristics such as decreased facial hair, testicular atrophy, diminished spermatogenesis, loss of libido, impotence, and decreased muscle mass are associated with decreases in follicle stimulating hormone (FSH) and luteinizing hormone (LH). Fasting hypoglycemia and hypotension occur in panhypopituitarism as a result of decreases in adrenocorticotropic hormone (ACTH) and cortisol. Bradycardia is likely due to the decrease in thyroid stimulating hormone (TSH) and thyroid hormones associated with panhypopituitarism.

11. Which patient action indicates good understanding of the nurse's teaching about administration of aspart (NovoLog) insulin? a. The patient avoids injecting the insulin into the upper abdominal area. b. The patient cleans the skin with soap and water before insulin administration. c. The patient stores the insulin in the freezer after administering the prescribed dose. d. The patient pushes the plunger down while removing the syringe from the injection site.

ANS: B Cleaning the skin with soap and water or with alcohol is acceptable. Insulin should not be frozen. The patient should leave the syringe in place for about 5 seconds after injection to be sure that all the insulin has been injected. The upper abdominal area is one of the preferred areas for insulin injection.

The nurse determines that demeclocycline (Declomycin) is effective for a patient with syndrome of inappropriate antidiuretic hormone (SIADH) based on finding that the patient's a. weight has increased. b. urinary output is increased. c. peripheral edema is decreased. d. urine specific gravity is increased.

ANS: B Demeclocycline blocks the action of antidiuretic hormone (ADH) on the renal tubules and increases urine output. An increase in weight or an increase in urine specific gravity indicates that the SIADH is not corrected. Peripheral edema does not occur with SIADH. A sudden weight gain without edema is a common clinical manifestation of this disorder.

The cardiac telemetry unit charge nurse receives status reports from other nursing units about four patients who need cardiac monitoring. Which patient should be transferred to the cardiac unit first? a. Patient with Hashimoto's thyroiditis and a heart rate of 102 b. Patient with tetany who has a new order for IV calcium chloride c. Patient with Cushing syndrome and a blood glucose of 140 mg/dL d. Patient with Addison's disease who takes hydrocortisone twice daily

ANS: B Emergency treatment of tetany requires IV administration of calcium

21. An unresponsive patient with type 2 diabetes is brought to the emergency department and diagnosed with hyperosmolar hyperglycemic syndrome (HHS). The nurse will anticipate the need to a. give a bolus of 50% dextrose. b. insert a large-bore IV catheter. c. initiate oxygen by nasal cannula. d. administer glargine (Lantus) insulin.

ANS: B HHS is initially treated with large volumes of IV fluids to correct hypovolemia. Regular insulin is administered, not a long-acting insulin. There is no indication that the patient requires oxygen. Dextrose solutions will increase the patient's blood glucose and would be contraindicated.

Which nursing assessment of a 69-year-old patient is most important to make during initiation of thyroid replacement with levothyroxine (Synthroid)? a. Fluid balance b. Apical pulse rate c. Nutritional intake d. Orientation and alertness

ANS: B In older patients, initiation of levothyroxine therapy can increase myocardial oxygen demand and cause angina or dysrhythmias. The medication also is expected to improve mental status and fluid balance and will increase metabolic rate and nutritional needs, but these changes will not result in potentially life-threatening complications.

After receiving change-of-shift report about the following four patients, which patient should the nurse assess first? a. A 31-year-old female with Cushing syndrome and a blood glucose level of 244 mg/dL b. A 70-year-old female taking levothyroxine (Synthroid) who has an irregular pulse of 134 c. A 53-year-old male who has Addison's disease and is due for a scheduled dose of hydrocortisone (Solu-Cortef). d. A 22-year-old male admitted with syndrome of inappropriate antidiuretic hormone (SIADH) who has a serum sodium level of 130 mEq/L

ANS: B Initiation of thyroid replacement in older adults may cause angina and cardiac dysrhythmias. The patient's high pulse rate needs rapid investigation by the nurse to assess for and intervene with any cardiac problems. The other patients also require nursing assessment and/or actions but are not at risk for life-threatening complications.

Which question will the nurse in the endocrine clinic ask to help determine a patient's risk factors for goiter? a. "How much milk do you drink?" b. "What medications are you taking?" c. "Are your immunizations up to date?" d. "Have you had any recent neck injuries?"

ANS: B Medications that contain thyroid-inhibiting substances can cause goiter. Milk intake, neck injury, and immunization history are not risk factors for goiter.

A patient who had radical neck surgery to remove a malignant tumor developed hypoparathyroidism. The nurse should plan to teach the patient about a. bisphosphonates to reduce bone demineralization. b. calcium supplements to normalize serum calcium levels. c. increasing fluid intake to decrease risk for nephrolithiasis. d. including whole grains in the diet to prevent constipation.

ANS: B Oral calcium supplements are used to maintain the serum calcium in normal range and prevent the complications of hypocalcemia. Whole grain foods decrease calcium absorption and will not be recommended. Bisphosphonates will lower serum calcium levels further by preventing calcium from being reabsorbed from bone. Kidney stones are not a complication of hypoparathyroidism and low calcium levels.

33. The nurse is taking a health history from a 29-year-old pregnant patient at the first prenatal visit. The patient reports no personal history of diabetes but has a parent who is diabetic. Which action will the nurse plan to take first? a. Teach the patient about administering regular insulin. b. Schedule the patient for a fasting blood glucose level. c. Discuss an oral glucose tolerance test for the twenty-fourth week of pregnancy. d. Provide teaching about an increased risk for fetal problems with gestational diabetes.

ANS: B Patients at high risk for gestational diabetes should be screened for diabetes on the initial prenatal visit. An oral glucose tolerance test may also be used to check for diabetes, but it would be done before the twenty-fourth week. The other actions may also be needed (depending on whether the patient develops gestational diabetes), but they are not the first actions that the nurse should take.

24. Which action should the nurse take after a 36-year-old patient treated with intramuscular glucagon for hypoglycemia regains consciousness? a. Assess the patient for symptoms of hyperglycemia. b. Give the patient a snack of peanut butter and crackers. c. Have the patient drink a glass of orange juice or nonfat milk. d. Administer a continuous infusion of 5% dextrose for 24 hours.

ANS: B Rebound hypoglycemia can occur after glucagon administration, but having a meal containing complex carbohydrates plus protein and fat will help prevent hypoglycemia. Orange juice and nonfat milk will elevate blood glucose rapidly, but the cheese and crackers will stabilize blood glucose. Administration of IV glucose might be used in patients who were unable to take in nutrition orally. The patient should be assessed for symptoms of hypoglycemia after glucagon administration.

Which information will the nurse include when teaching a 50-year-old male patient about somatropin (Genotropin)? a. The medication will be needed for 3 to 6 months. b. Inject the medication subcutaneously every day. c. Blood glucose levels may decrease when taking the medication. d. Stop taking the medication if swelling of the hands or feet occurs.

ANS: B Somatropin is injected subcutaneously on a daily basis, preferably in the evening. The patient will need to continue on somatropin for life. If swelling or other common adverse effects occur, the health care provider should be notified. Growth hormone will increase blood glucose levels.

Which intervention will the nurse include in the plan of care for a 52-year-old male patient with syndrome of inappropriate antidiuretic hormone (SIADH)? a. Monitor for peripheral edema. b. Offer patient hard candies to suck on. c. Encourage fluids to 2 to 3 liters per day. d. Keep head of bed elevated to 30 degrees.

ANS: B Sucking on hard candies decreases thirst for a patient on fluid restriction. Patients with SIADH are on fluid restrictions of 800 to 1000 mL/day. Peripheral edema is not seen with SIADH. The head of the bed is elevated no more than 10 degrees to increase left atrial filling pressure and decrease antidiuretic hormone (ADH) release.

The nurse will plan to monitor a patient diagnosed with a pheochromocytoma for a. flushing. b. headache. c. bradycardia. d. hypoglycemia.

ANS: B The classic clinical manifestations of pheochromocytoma are hypertension, tachycardia, severe headache, diaphoresis, and abdominal or chest pain. Elevated blood glucose may also occur because of sympathetic nervous system stimulation. Bradycardia and flushing would not be expected.

A 37-year-old patient has just arrived in the postanesthesia recovery unit (PACU) after a thyroidectomy. Which information is most important to communicate to the surgeon? a. The patient reports 7/10 incisional pain. b. The patient has increasing neck swelling. c. The patient is sleepy and difficult to arouse. d. The patient's cardiac rate is 112 beats/minute.

ANS: B The neck swelling may lead to respiratory difficulty, and rapid intervention is needed to prevent airway obstruction. The incisional pain should be treated but is not unusual after surgery. A heart rate of 112 is not unusual in a patient who has been hyperthyroid and has just arrived in the PACU from surgery. Sleepiness in the immediate postoperative period is expected.

A 38-year-old male patient is admitted to the hospital in Addisonian crisis. Which patient statement supports a nursing diagnosis of ineffective self-health management related to lack of knowledge about management of Addison's disease? a. "I frequently eat at restaurants, and my food has a lot of added salt." b. "I had the stomach flu earlier this week, so I couldn't take the hydrocortisone." c. "I always double my dose of hydrocortisone on the days that I go for a long run." d. "I take twice as much hydrocortisone in the morning dose as I do in the afternoon."

ANS: B The need for hydrocortisone replacement is increased with stressors such as illness, and the patient needs to be taught to call the health care provider because medication and IV fluids and electrolytes may need to be given. The other patient statements indicate appropriate management of the Addison's disease.

38. Which laboratory value reported to the nurse by the unlicensed assistive personnel (UAP) indicates the most urgent need for the nurse's assessment of the patient? a. Bedtime glucose of 140 mg/dL b. Noon blood glucose of 52 mg/dL c. Fasting blood glucose of 130 mg/dL d. 2-hr postprandial glucose of 220 mg/dL

ANS: B The nurse should assess the patient with a blood glucose level of 52 mg/dL for symptoms of hypoglycemia and give the patient a carbohydrate-containing beverage such as orange juice. The other values are within an acceptable range or not immediately dangerous for a diabetic patient.

19. The nurse identifies a need for additional teaching when the patient who is self-monitoring blood glucose a. washes the puncture site using warm water and soap. b. chooses a puncture site in the center of the finger pad. c. hangs the arm down for a minute before puncturing the site. d. says the result of 120 mg indicates good blood sugar control.

ANS: B The patient is taught to choose a puncture site at the side of the finger pad because there are fewer nerve endings along the side of the finger pad. The other patient actions indicate that teaching has been effective.

Which nursing action will be included in the plan of care for a 55-year-old patient with Graves' disease who has exophthalmos? a. Place cold packs on the eyes to relieve pain and swelling. b. Elevate the head of the patient's bed to reduce periorbital fluid. c. Apply alternating eye patches to protect the corneas from irritation. d. Teach the patient to blink every few seconds to lubricate the corneas.

ANS: B The patient should sit upright as much as possible to promote fluid drainage from the periorbital area. With exophthalmos, the patient is unable to close the eyes completely to blink. Lubrication of the eyes, rather than eye patches, will protect the eyes from developing corneal scarring. The swelling of the eye is not caused by excessive blood flow to the eye, so cold packs will not be helpful.

A 63-year-old patient with primary hyperparathyroidism has a serum phosphorus level of 1.7 mg/dL (0.55 mmol/L) and calcium of 14 mg/dL (3.5 mmol/L). Which nursing action should be included in the plan of care? a. Restrict the patient to bed rest. b. Encourage 4000 mL of fluids daily. c. Institute routine seizure precautions. d. Assess for positive Chvostek's sign.

ANS: B The patient with hypercalcemia is at risk for kidney stones, which may be prevented by a high fluid intake. Seizure precautions and monitoring for Chvostek's or Trousseau's sign are appropriate for hypocalcemic patients. The patient should engage in weight-bearing exercise to decrease calcium loss from bone.

A patient who had a subtotal thyroidectomy earlier today develops laryngeal stridor and a cramp in the right hand upon returning to the surgical nursing unit. Which collaborative action will the nurse anticipate next? a. Suction the patient's airway. b. Administer IV calcium gluconate. c. Plan for emergency tracheostomy. d. Prepare for endotracheal intubation.

ANS: B The patient's clinical manifestations of stridor and cramping are consistent with tetany caused by hypocalcemia resulting from damage to the parathyroid glands during surgery. Endotracheal intubation or tracheostomy may be needed if the calcium does not resolve the stridor. Suctioning will not correct the stridor.

15. Which information will the nurse include when teaching a 50-year-old patient who has type 2 diabetes about glyburide (Micronase, DiaBeta, Glynase)? a. Glyburide decreases glucagon secretion from the pancreas. b. Glyburide stimulates insulin production and release from the pancreas. c. Glyburide should be taken even if the morning blood glucose level is low. d. Glyburide should not be used for 48 hours after receiving IV contrast media.

ANS: B The sulfonylureas stimulate the production and release of insulin from the pancreas. If the glucose level is low, the patient should contact the health care provider before taking the glyburide, because hypoglycemia can occur with this class of medication. Metformin should be held for 48 hours after administration of IV contrast media, but this is not necessary for glyburide. Glucagon secretion is not affected by glyburide.

40. An active 28-year-old male with type 1 diabetes is being seen in the endocrine clinic. Which finding may indicate the need for a change in therapy? a. Hemoglobin A1C level 6.2% b. Blood pressure 146/88 mmHg c. Heart rate at rest 58 beats/minute d. High density lipoprotein (HDL) level 65 mg/dL

ANS: B To decrease the incidence of macrovascular and microvascular problems in patients with diabetes, the goal blood pressure is usually 130/80. An A1C less than 6.5%, a low resting heart rate (consistent with regular aerobic exercise in a young adult), and an HDL level of 65 mg/dL all indicate that the patient's diabetes and risk factors for vascular disease are well controlled.

28. A diabetic patient who has reported burning foot pain at night receives a new prescription. Which information should the nurse teach the patient about amitriptyline (Elavil)? a. Amitriptyline decreases the depression caused by your foot pain. b. Amitriptyline helps prevent transmission of pain impulses to the brain. c. Amitriptyline corrects some of the blood vessel changes that cause pain. d. Amitriptyline improves sleep and makes you less aware of nighttime pain.

ANS: B Tricyclic antidepressants decrease the transmission of pain impulses to the spinal cord and brain. Tricyclic antidepressants also improve sleep quality and are used for depression, but that is not the major purpose for their use in diabetic neuropathy. The blood vessel changes that contribute to neuropathy are not affected by tricyclic antidepressants.

A 56-year-old patient who is disoriented and reports a headache and muscle cramps is hospitalized with possible syndrome of inappropriate antidiuretic hormone (SIADH). The nurse would expect the initial laboratory results to include a(n) a. elevated hematocrit. b. decreased serum sodium. c. low urine specific gravity. d. increased serum chloride.

ANS: B When water is retained, the serum sodium level will drop below normal, causing the clinical manifestations reported by the patient. The hematocrit will decrease because of the dilution caused by water retention. Urine will be more concentrated with a higher specific gravity. The serum chloride level will usually decrease along with the sodium level.

The patient with diabetes insipidus is brought to the emergency department with confusion and dehydration after excretion of a large volume of urine today even though several liters of fluid were drunk. What is a diagnostic test that the nurse should expect to be done to help make a diagnosis? a. Blood glucose c. Urine specific gravity b. Serum sodium level d. Computed tomography (CT) of the head

c. Patients with diabetes insipidus (DI) excrete large amounts of urine with a specific gravity of less than 1.005. Blood glucose would be tested to diagnose diabetes mellitus. The serum sodium level is expected to be low with DI but is not diagnostic. To diagnose central DI a water deprivation test is required. Then a CT of the head may be done to determine the cause. Nephrogenic DI is differentiated from central DI with determination of the level of ADH after an analog of ADH is given.

1. To monitor for complications in a patient with type 2 diabetes, which tests will the nurse in the diabetic clinic schedule at least annually (select all that apply)? a. Chest x-ray b. Blood pressure c. Serum creatinine d. Urine for microalbuminuria e. Complete blood count (CBC) f. Monofilament testing of the foot

ANS: B, C, D, F Blood pressure, serum creatinine, urine testing for microalbuminuria, and monofilament testing of the foot are recommended at least annually to screen for possible microvascular and macrovascular complications of diabetes. Chest x-ray and CBC might be ordered if the diabetic patient presents with symptoms of respiratory or infectious problems but are not routinely included in screening.

A 29-year-old woman with systemic lupus erythematosus has been prescribed 2 weeks of high-dose prednisone therapy. Which information about the prednisone is most important for the nurse to include? a. "Weigh yourself daily to monitor for weight gain caused by increased appetite." b. "A weight-bearing exercise program will help minimize the risk for osteoporosis." c. "The prednisone dose should be decreased gradually rather than stopped suddenly." d. "Call the health care provider if you experience mood alterations with the prednisone."

ANS: C Acute adrenal insufficiency may occur if exogenous corticosteroids are suddenly stopped. Mood alterations and weight gain are possible adverse effects of corticosteroid use, but these are not life-threatening effects. Osteoporosis occurs when patients take corticosteroids for longer periods.

The nurse is caring for a patient following an adrenalectomy. The highest priority in the immediate postoperative period is to a. protect the patient's skin. b. monitor for signs of infection. c. balance fluids and electrolytes. d. prevent emotional disturbances.

ANS: C After adrenalectomy, the patient is at risk for circulatory instability caused by fluctuating hormone levels, and the focus of care is to assess and maintain fluid and electrolyte status through the use of IV fluids and corticosteroids. The other goals are also important for the patient but are not as immediately life threatening as the circulatory collapse that can occur with fluid and electrolyte disturbances.

A patient has just arrived on the unit after a thyroidectomy. Which action should the nurse take first? a. Observe the dressing for bleeding. b. Check the blood pressure and pulse. c. Assess the patient's respiratory effort. d. Support the patient's head with pillows.

ANS: C Airway obstruction is a possible complication after thyroidectomy because of swelling or bleeding at the site or tetany. The priority nursing action is to assess the airway. The other actions are also part of the standard nursing care postthyroidectomy but are not as high of a priority.

After a 22-year-old female patient with a pituitary adenoma has had a hypophysectomy, the nurse will teach about the need for a. sodium restriction to prevent fluid retention. b. insulin to maintain normal blood glucose levels. c. oral corticosteroids to replace endogenous cortisol. d. chemotherapy to prevent malignant tumor recurrence.

ANS: C Antidiuretic hormone (ADH), cortisol, and thyroid hormone replacement will be needed for life after hypophysectomy. Without the effects of adrenocorticotropic hormone (ACTH) and cortisol, the blood glucose and serum sodium will be low unless cortisol is replaced. An adenoma is a benign tumor, and chemotherapy will not be needed.

1. Which statement by a nurse to a patient newly diagnosed with type 2 diabetes is correct? a. Insulin is not used to control blood glucose in patients with type 2 diabetes. b. Complications of type 2 diabetes are less serious than those of type 1 diabetes. c. Changes in diet and exercise may control blood glucose levels in type 2 diabetes. d. Type 2 diabetes is usually diagnosed when the patient is admitted with a hyperglycemic coma.

ANS: C For some patients with type 2 diabetes, changes in lifestyle are sufficient to achieve blood glucose control. Insulin is frequently used for type 2 diabetes, complications are equally severe as for type 1 diabetes, and type 2 diabetes is usually diagnosed with routine laboratory testing or after a patient develops complications such as frequent yeast infections.

17. When a patient who takes metformin (Glucophage) to manage type 2 diabetes develops an allergic rash from an unknown cause, the health care provider prescribes prednisone (Deltasone). The nurse will anticipate that the patient may a. need a diet higher in calories while receiving prednisone. b. develop acute hypoglycemia while taking the prednisone. c. require administration of insulin while taking prednisone. d. have rashes caused by metformin-prednisone interactions.

ANS: C Glucose levels increase when patients are taking corticosteroids, and insulin may be required to control blood glucose. Hypoglycemia is not a side effect of prednisone. Rashes are not an adverse effect caused by taking metformin and prednisone simultaneously. The patient may have an increased appetite when taking prednisone, but will not need a diet that is higher in calories.

23. The health care provider suspects the Somogyi effect in a 50-year-old patient whose 6:00 AM blood glucose is 230 mg/dL. Which action will the nurse teach the patient to take? a. Avoid snacking at bedtime. b. Increase the rapid-acting insulin dose. c. Check the blood glucose during the night d. Administer a larger dose of long-acting insulin.

ANS: C If the Somogyi effect is causing the patient's increased morning glucose level, the patient will experience hypoglycemia between 2:00 and 4:00 AM. The dose of insulin will be reduced, rather than increased. A bedtime snack is used to prevent hypoglycemic episodes during the night.

22. A 26-year-old female with type 1 diabetes develops a sore throat and runny nose after caring for her sick toddler. The patient calls the clinic for advice about her symptoms and a blood glucose level of 210 mg/dL despite taking her usual glargine (Lantus) and lispro (Humalog) insulin. The nurse advises the patient to a. use only the lispro insulin until the symptoms are resolved. b. limit intake of calories until the glucose is less than 120 mg/dL. c. monitor blood glucose every 4 hours and notify the clinic if it continues to rise. d. decrease intake of carbohydrates until glycosylated hemoglobin is less than 7%.

ANS: C Infection and other stressors increase blood glucose levels and the patient will need to test blood glucose frequently, treat elevations appropriately with lispro insulin, and call the health care provider if glucose levels continue to be elevated. Discontinuing the glargine will contribute to hyperglycemia and may lead to diabetic ketoacidosis (DKA). Decreasing carbohydrate or caloric intake is not appropriate because the patient will need more calories when ill. Glycosylated hemoglobin testing is not used to evaluate short-term alterations in blood glucose.

39. When a patient with type 2 diabetes is admitted for a cholecystectomy, which nursing action can the nurse delegate to a licensed practical/vocational nurse (LPN/LVN)? a. Communicate the blood glucose level and insulin dose to the circulating nurse in surgery. b. Discuss the reason for the use of insulin therapy during the immediate postoperative period. c. Administer the prescribed lispro (Humalog) insulin before transporting the patient to surgery. d. Plan strategies to minimize the risk for hypoglycemia or hyperglycemia during the postoperative period.

ANS: C LPN/LVN education and scope of practice includes administration of insulin. Communication about patient status with other departments, planning, and patient teaching are skills that require RN education and scope of practice.

Which information will the nurse teach a 48-year-old patient who has been newly diagnosed with Graves' disease? a. Exercise is contraindicated to avoid increasing metabolic rate. b. Restriction of iodine intake is needed to reduce thyroid activity. c. Antithyroid medications may take several months for full effect. d. Surgery will eventually be required to remove the thyroid gland.

ANS: C Medications used to block the synthesis of thyroid hormones may take 2 to 3 months before the full effect is seen. Large doses of iodine are used to inhibit the synthesis of thyroid hormones. Exercise using large muscle groups is encouraged to decrease the irritability and hyperactivity associated with high levels of thyroid hormones. Radioactive iodine is the most common treatment for Graves' disease although surgery may be used.

8. The nurse determines a need for additional instruction when the patient with newly diagnosed type 1 diabetes says which of the following? a. "I can have an occasional alcoholic drink if I include it in my meal plan." b. "I will need a bedtime snack because I take an evening dose of NPH insulin." c. "I can choose any foods, as long as I use enough insulin to cover the calories." d. "I will eat something at meal times to prevent hypoglycemia, even if I am not hungry."

ANS: C Most patients with type 1 diabetes need to plan diet choices very carefully. Patients who are using intensified insulin therapy have considerable flexibility in diet choices but still should restrict dietary intake of items such as fat, protein, and alcohol. The other patient statements are correct and indicate good understanding of the diet instruction.

An expected nursing diagnosis for a 30-year-old patient admitted to the hospital with symptoms of diabetes insipidus is a. excess fluid volume related to intake greater than output. b. impaired gas exchange related to fluid retention in lungs. c. sleep pattern disturbance related to frequent waking to void. d. risk for impaired skin integrity related to generalized edema.

ANS: C Nocturia occurs as a result of the polyuria caused by diabetes insipidus. Edema, excess fluid volume, and fluid retention are not expected.

31. A 26-year-old patient with diabetes rides a bicycle to and from work every day. Which site should the nurse teach the patient to administer the morning insulin? a. thigh. b. buttock. c. abdomen. d. upper arm.

ANS: C Patients should be taught not to administer insulin into a site that will be exercised because exercise will increase the rate of absorption. The thigh, buttock, and arm are all exercised by riding a bicycle.

A 37-year-old patient is being admitted with a diagnosis of Cushing syndrome. Which findings will the nurse expect during the assessment? a. Chronically low blood pressure b. Bronzed appearance of the skin c. Purplish streaks on the abdomen d. Decreased axillary and pubic hair

ANS: C Purplish-red striae on the abdomen are a common clinical manifestation of Cushing syndrome. Hypotension and bronzed-appearing skin are manifestations of Addison's disease. Decreased axillary and pubic hair occur with androgen deficiency.

42. After the nurse has finished teaching a patient who has a new prescription for exenatide (Byetta), which patient statement indicates that the teaching has been effective? a. "I may feel hungrier than usual when I take this medicine." b. "I will not need to worry about hypoglycemia with the Byetta." c. "I should take my daily aspirin at least an hour before the Byetta." d. "I will take the pill at the same time I eat breakfast in the morning."

ANS: C Since exenatide slows gastric emptying, oral medications should be taken at least an hour before the exenatide to avoid slowing absorption. Exenatide is injected and increases feelings of satiety. Hypoglycemia can occur with this medication.

Which information obtained by the nurse in the endocrine clinic about a patient who has been taking prednisone (Deltasone) 40 mg daily for 3 weeks is most important to report to the health care provider? a. Patient's blood pressure is 148/94 mm Hg. b. Patient has bilateral 2+ pitting ankle edema. c. Patient stopped taking the medication 2 days ago. d. Patient has not been taking the prescribed vitamin D.

ANS: C Sudden cessation of corticosteroids after taking the medication for a week or more can lead to adrenal insufficiency, with problems such as severe hypotension and hypoglycemia. The patient will need immediate evaluation by the health care provider to prevent and/or treat adrenal insufficiency. The other information will also be reported, but does not require rapid treatment.

43. A few weeks after an 82-year-old with a new diagnosis of type 2 diabetes has been placed on metformin (Glucophage) therapy and taught about appropriate diet and exercise, the home health nurse makes a visit. Which finding by the nurse is most important to discuss with the health care provider? a. Hemoglobin A1C level is 7.9%. b. Last eye exam was 18 months ago. c. Glomerular filtration rate is decreased. d. Patient has questions about the prescribed diet.

ANS: C The decrease in renal function may indicate a need to adjust the dose of metformin or change to a different medication. In older patients, the goal for A1C may be higher in order to avoid complications associated with hypoglycemia. The nurse will plan on scheduling the patient for an eye exam and addressing the questions about diet, but the biggest concern is the patient's decreased renal function.

Which assessment finding of a 42-year-old patient who had a bilateral adrenalectomy requires the most rapid action by the nurse? a. The blood glucose is 176 mg/dL. b. The lungs have bibasilar crackles. c. The blood pressure (BP) is 88/50 mm Hg. d. The patient reports 5/10 incisional pain.

ANS: C The decreased BP indicates possible adrenal insufficiency. The nurse should immediately notify the health care provider so that corticosteroid medications can be administered. The nurse should also address the elevated glucose, incisional pain, and crackles with appropriate collaborative or nursing actions, but prevention and treatment of acute adrenal insufficiency is the priority after adrenalectomy.

36. A patient who was admitted with diabetic ketoacidosis secondary to a urinary tract infection has been weaned off an insulin drip 30 minutes ago. The patient reports feeling lightheaded and sweaty. Which action should the nurse take first? a. Infuse dextrose 50% by slow IV push. b. Administer 1 mg glucagon subcutaneously. c. Obtain a glucose reading using a finger stick. d. Have the patient drink 4 ounces of orange juice.

ANS: C The patient's clinical manifestations are consistent with hypoglycemia and the initial action should be to check the patient's glucose with a finger stick or order a stat blood glucose. If the glucose is low, the patient should ingest a rapid-acting carbohydrate, such as orange juice. Glucagon or dextrose 50% might be given if the patient's symptoms become worse or if the patient is unconscious.

46. After change-of-shift report, which patient will the nurse assess first? a. 19-year-old with type 1 diabetes who was admitted with possible dawn phenomenon b. 35-year-old with type 1 diabetes whose most recent blood glucose reading was 230 mg/dL c. 60-year-old with hyperosmolar hyperglycemic syndrome who has poor skin turgor and dry oral mucosa d. 68-year-old with type 2 diabetes who has severe peripheral neuropathy and complains of burning foot pain

ANS: C The patient's diagnosis of HHS and signs of dehydration indicate that the nurse should rapidly assess for signs of shock and determine whether increased fluid infusion is needed. The other patients also need assessment and intervention but do not have life-threatening complications.

2. A 48-year-old male patient screened for diabetes at a clinic has a fasting plasma glucose level of 120 mg/dL (6.7 mmol/L). The nurse will plan to teach the patient about a. self-monitoring of blood glucose. b. using low doses of regular insulin. c. lifestyle changes to lower blood glucose. d. effects of oral hypoglycemic medications.

ANS: C The patient's impaired fasting glucose indicates prediabetes, and the patient should be counseled about lifestyle changes to prevent the development of type 2 diabetes. The patient with prediabetes does not require insulin or oral hypoglycemics for glucose control and does not need to self-monitor blood glucose.

A patient develops carpopedal spasms and tingling of the lips following a parathyroidectomy. Which action should the nurse take first? a. Administer the ordered muscle relaxant. b. Give the ordered oral calcium supplement. c. Have the patient rebreathe from a paper bag. d. Start the PRN oxygen at 2 L/min per cannula.

ANS: C The patient's symptoms suggest mild hypocalcemia. The symptoms of hypocalcemia will be temporarily reduced by having the patient breathe into a paper bag, which will raise the PaCO2 and create a more acidic pH. The muscle relaxant will have no impact on the ionized calcium level. Although severe hypocalcemia can cause laryngeal stridor, there is no indication that this patient is experiencing laryngeal stridor or needs oxygen. Calcium supplements will be given to normalize calcium levels quickly, but oral supplements will take time to be absorbed.

Which assessment finding for a 33-year-old female patient admitted with Graves' disease requires the most rapid intervention by the nurse? a. Bilateral exophthalmos b. Heart rate 136 beats/minute c. Temperature 103.8° F (40.4° C) d. Blood pressure 166/100 mm Hg

ANS: C The patient's temperature indicates that the patient may have thyrotoxic crisis and that interventions to lower the temperature are needed immediately. The other findings also require intervention but do not indicate potentially life-threatening complications.

A 62-year-old patient with hyperthyroidism is to be treated with radioactive iodine (RAI). The nurse instructs the patient a. about radioactive precautions to take with all body secretions. b. that symptoms of hyperthyroidism should be relieved in about a week. c. that symptoms of hypothyroidism may occur as the RAI therapy takes effect. d. to discontinue the antithyroid medications taken before the radioactive therapy.

ANS: C There is a high incidence of postradiation hypothyroidism after RAI, and the patient should be monitored for symptoms of hypothyroidism. RAI has a delayed response, with the maximum effect not seen for 2 to 3 months, and the patient will continue to take antithyroid medications during this time. The therapeutic dose of radioactive iodine is low enough that no radiation safety precautions are needed.

A 42-year-old female patient is scheduled for transsphenoidal hypophysectomy to treat a pituitary adenoma. During preoperative teaching, the nurse instructs the patient about the need to a. cough and deep breathe every 2 hours postoperatively. b. remain on bed rest for the first 48 hours after the surgery. c. avoid brushing teeth for at least 10 days after the surgery. d. be positioned flat with sandbags at the head postoperatively.

ANS: C To avoid disruption of the suture line, the patient should avoid brushing the teeth for 10 days after surgery. It is not necessary to remain on bed rest after this surgery. Coughing is discouraged because it may cause leakage of cerebrospinal fluid (CSF) from the suture line. The head of the bed should be elevated 30 degrees to reduce pressure on the sella turcica and decrease the risk for headaches.

29. Which information is most important for the nurse to report to the health care provider before a patient with type 2 diabetes is prepared for a coronary angiogram? a. The patient's most recent HbA1C was 6.5%. b. The patient's admission blood glucose is 128 mg/dL. c. The patient took the prescribed metformin (Glucophage) today. d. The patient took the prescribed captopril (Capoten) this morning.

ANS: C To avoid lactic acidosis, metformin should be discontinued a day or 2 before the coronary arteriogram and should not be used for 48 hours after IV contrast media are administered. The other patient data will also be reported but do not indicate any need to reschedule the procedure.

4. The nurse is assessing a 22-year-old patient experiencing the onset of symptoms of type 1 diabetes. Which question is most appropriate for the nurse to ask? a. "Are you anorexic?" b. "Is your urine dark colored?" c. "Have you lost weight lately?" d. "Do you crave sugary drinks?"

ANS: C Weight loss occurs because the body is no longer able to absorb glucose and starts to break down protein and fat for energy. The patient is thirsty but does not necessarily crave sugar-containing fluids. Increased appetite is a classic symptom of type 1 diabetes. With the classic symptom of polyuria, urine will be very dilute.

An 82-year-old patient in a long-term care facility has several medications prescribed. After the patient is newly diagnosed with hypothyroidism, the nurse will need to consult with the health care provider before administering a. docusate (Colace). b. ibuprofen (Motrin). c. diazepam (Valium). d. cefoxitin (Mefoxin).

ANS: C Worsening of mental status and myxedema coma can be precipitated by the use of sedatives, especially in older adults. The nurse should discuss the use of diazepam with the health care provider before administration. The other medications may be given safely to the patient.

Which information is most important for the nurse to communicate rapidly to the health care provider about a patient admitted with possible syndrome of inappropriate antidiuretic hormone (SIADH)? a. The patient has a recent weight gain of 9 lb. b. The patient complains of dyspnea with activity. c. The patient has a urine specific gravity of 1.025. d. The patient has a serum sodium level of 118 mEq/L.

ANS: D A serum sodium of less than 120 mEq/L increases the risk for complications such as seizures and needs rapid correction. The other data are not unusual for a patient with SIADH and do not indicate the need for rapid action

A 23-year-old patient is admitted with diabetes insipidus. Which action will be most appropriate for the registered nurse (RN) to delegate to an experienced licensed practical/vocational nurse (LPN/LVN)? a. Titrate the infusion of 5% dextrose in water. b. Teach the patient how to use desmopressin (DDAVP) nasal spray. c. Assess the patient's hydration status every 8 hours. d. Administer subcutaneous DDAVP.

ANS: D Administration of medications is included in LPN/LVN education and scope of practice. Assessments, patient teaching, and titrating fluid infusions are more complex skills and should be done by the RN.

18. A hospitalized diabetic patient received 38 U of NPH insulin at 7:00 AM. At 1:00 PM, the patient has been away from the nursing unit for 2 hours, missing the lunch delivery while awaiting a chest x-ray. To prevent hypoglycemia, the best action by the nurse is to a. save the lunch tray for the patient's later return to the unit. b. ask that diagnostic testing area staff to start a 5% dextrose IV. c. send a glass of milk or orange juice to the patient in the diagnostic testing area. d. request that if testing is further delayed, the patient be returned to the unit to eat.

ANS: D Consistency for mealtimes assists with regulation of blood glucose, so the best option is for the patient to have lunch at the usual time. Waiting to eat until after the procedure is likely to cause hypoglycemia. Administration of an IV solution is unnecessarily invasive for the patient. A glass of milk or juice will keep the patient from becoming hypoglycemic but will cause a rapid rise in blood glucose because of the rapid absorption of the simple carbohydrate in these items.

30. Which action by a patient indicates that the home health nurse's teaching about glargine and regular insulin has been successful? a. The patient administers the glargine 30 minutes before each meal. b. The patient's family prefills the syringes with the mix of insulins weekly. c. The patient draws up the regular insulin and then the glargine in the same syringe. d. The patient disposes of the open vials of glargine and regular insulin after 4 weeks.

ANS: D Insulin can be stored at room temperature for 4 weeks. Glargine should not be mixed with other insulins or prefilled and stored. Short-acting regular insulin is administered before meals, while glargine is given once daily.

Which finding by the nurse when assessing a patient with a large pituitary adenoma is most important to report to the health care provider? a. Changes in visual field b. Milk leaking from breasts c. Blood glucose 150 mg/dL d. Nausea and projectile vomiting

ANS: D Nausea and projectile vomiting may indicate increased intracranial pressure, which will require rapid actions for diagnosis and treatment. Changes in the visual field, elevated blood glucose, and galactorrhea are common with pituitary adenoma, but these do not require rapid action to prevent life-threatening complications.

32. The nurse is interviewing a new patient with diabetes who receives rosiglitazone (Avandia) through a restricted access medication program. What is most important for the nurse to report immediately to the health care provider? a. The patient's blood pressure is 154/92. b. The patient has a history of emphysema. c. The patient's blood glucose is 86 mg/dL. d. The patient has chest pressure when walking.

ANS: D Rosiglitazone can cause myocardial ischemia. The nurse should immediately notify the health care provider and expect orders to discontinue the medication. There is no urgent need to discuss the other data with the health care provider.

27. Which finding indicates a need to contact the health care provider before the nurse administers metformin (Glucophage)? a. The patient's blood glucose level is 174 mg/dL. b. The patient has gained 2 lb (0.9 kg) since yesterday. c. The patient is scheduled for a chest x-ray in an hour. d. The patient's blood urea nitrogen (BUN) level is 52 mg/dL.

ANS: D The BUN indicates possible renal failure, and metformin should not be used in patients with renal failure. The other findings are not contraindications to the use of metformin.

5. A patient with type 2 diabetes is scheduled for a follow-up visit in the clinic several months from now. Which test will the nurse schedule to evaluate the effectiveness of treatment for the patient? a. Urine dipstick for glucose b. Oral glucose tolerance test c. Fasting blood glucose level d. Glycosylated hemoglobin level

ANS: D The glycosylated hemoglobin (A1C or HbA1C) test shows the overall control of glucose over 90 to 120 days. A fasting blood level indicates only the glucose level at one time. Urine glucose testing is not an accurate reflection of blood glucose level and does not reflect the glucose over a prolonged time. Oral glucose tolerance testing is done to diagnose diabetes, but is not used for monitoring glucose control once diabetes has been diagnosed.

A 44-year-old female patient with Cushing syndrome is admitted for adrenalectomy. Which intervention by the nurse will be most helpful for a nursing diagnosis of disturbed body image related to changes in appearance? a. Reassure the patient that the physical changes are very common in patients with Cushing syndrome. b. Discuss the use of diet and exercise in controlling the weight gain associated with Cushing syndrome. c. Teach the patient that the metabolic impact of Cushing syndrome is of more importance than appearance. d. Remind the patient that most of the physical changes caused by Cushing syndrome will resolve after surgery.

ANS: D The most reassuring communication to the patient is that the physical and emotional changes caused by the Cushing syndrome will resolve after hormone levels return to normal postoperatively. Reassurance that the physical changes are expected or that there are more serious physiologic problems associated with Cushing syndrome are not therapeutic responses. The patient's physiological changes are caused by the high hormone levels, not by the patient's diet or exercise choices.

16. The nurse has been teaching a patient with type 2 diabetes about managing blood glucose levels and taking glipizide (Glucotrol). Which patient statement indicates a need for additional teaching? a. "If I overeat at a meal, I will still take the usual dose of medication." b. "Other medications besides the Glucotrol may affect my blood sugar." c. "When I am ill, I may have to take insulin to control my blood sugar." d. "My diabetes won't cause complications because I don't need insulin."

ANS: D The patient should understand that type 2 diabetes places the patient at risk for many complications and that good glucose control is as important when taking oral agents as when using insulin. The other statements are accurate and indicate good understanding of the use of glipizide.

3. A 28-year-old male patient with type 1 diabetes reports how he manages his exercise and glucose control. Which behavior indicates that the nurse should implement additional teaching? a. The patient always carries hard candies when engaging in exercise. b. The patient goes for a vigorous walk when his glucose is 200 mg/dL. c. The patient has a peanut butter sandwich before going for a bicycle ride. d. The patient increases daily exercise when ketones are present in the urine.

ANS: D When the patient is ketotic, exercise may result in an increase in blood glucose level. Type 1 diabetic patients should be taught to avoid exercise when ketosis is present. The other statements are correct.

Type 1 Diabetes Mellitus - Endogenous insulin

Absent

Type 2 Diabetes Mellitus - Islet cell antibodies

Absent

Type 1 Diabetes Mellitus - Primary defect

Absent or minimal insulin production

Type 1 Diabetes Mellitus - Prevalence

Accounts for 5%-10% of all types of diabetes

Type 2 Diabetes Mellitus - Prevalence

Accounts for 90%-95% of all types of diabetes

Bacterial or fungal infection of thyroid gland

Acute thyroiditis

Leading cause of...

Adult blindness End-stage renal disease Nontraumatic lower limb amputations

Type 2 diabetes is more prevalent in...

African Americans, Asian Americans, Hispanics, Native Hawaiians or other Pacific Islanders, and Native Americans have a higher rate of type 2 diabetes than whites.

1 The clot retraction is the reflection of the time it takes to retract or shrink the clot from the sides of the test tube. This normally takes between 1 and 24 hours. However, the patient has a clot retraction time of 48 hours, indicating abnormal platelet function. The normal thrombin time is 17 to 23 seconds and the patient has a thrombin time of 22 seconds. This indicates that the patient has normal thrombin activity. The normal prothrombin time is 11 to 16 seconds and the patient has a prothrombin time of 15 seconds. This indicates that the patient has normal results and no involvement of the extrinsic system. The activated clotting time indicates coagulation status and the normal activated clotting time is 70 to 120 seconds. The patient has an activated clotting time of 2 minutes, indicating that the patient has normal coagulation. Text Reference - p. 627

After assessing the laboratory reports of a patient, the nurse concludes that the patient has impaired platelet function. Which finding supports the nurse's conclusion? 1 A clot retraction of 48 hours 2 A thrombin time of 22 seconds 3 A prothrombin time of 15 seconds 4 An activated clotting time of 2 minutes

4 The nurse would ask the patient to sleep on the site of aspiration. This action reduces bleeding due to the application of pressure on the site. Warfarin is an anticoagulant, which increases bleeding. Therefore, the patient would have excess bleeding after taking warfarin. Ibuprofen is a nonsteroidal antiinflammatory drug used to help reduce pain. However, it is not effective in reducing bleeding. A soft bed will not apply as much pressure to the site. Hence, the nurse asks the patient to roll a towel and apply pressure to the site if sleeping on soft bed. Text Reference - p. 628

After performing the bone marrow aspiration, the nurse finds that the patient has bleeding at the site of aspiration. Which is the best nursing intervention in this situation? 1 Administering warfarin to the patient 2 Administering ibuprofen to the patient 3 Asking the patient to sleep on a soft bed 4 Asking the patient to sleep on the site for 30 minutes

2 The ileum is the site of cobalamin absorption, essential for proper functioning of the red blood cells. Therefore, the patient with an ileal resection will have a bleeding disorder due to impaired cobalamin absorption. The duodenum is the site for iron absorption. Therefore, the patient with a duodenal excision will have impaired iron absorption that results in a bleeding disorder. The patient with a partial or total gastrectomy will have a loss of parietal cells, which reduces the intrinsic factor levels. A patient who underwent a gastric bypass will have reduced parietal surface area, because the duodenum is bypassed. Text Reference - p. 620

After reviewing the medical records of a patient with a bleeding disorder, the nurse finds that the patient underwent an ileal resection. Which reason does the nurse suspect behind this finding? 1 Impaired iron absorption 2 Impaired cobalamin absorption 3 Decrease of intrinsic factor levels 4 Decrease of parietal cell surface area

however, aging does not affect erythropoietin secretion. Text Reference - p. 618

An elderly patient has low hemoglobin levels. The patient is otherwise healthy and has no signs of gastrointestinal bleeding. The nurse recognizes that what could be the reason for the low hemoglobin level? 1 Low level of hepicidin 2 Increase in iron-binding capacity 3 Decrease in intestinal absorption of iron 4 Decrease in erythropoietin secretion from the kidneys

2, 3 Elderly patients usually show only a minimal elevation in the total WBC count. It may be due to decreased T-cell function and humoral antibody response related to aging. The laboratory reports may not be erroneous, because infections in elderly patients do not manifest as a high WBC count. There may not be any error in obtaining the sample. The elderly patient usually has a decreased bone marrow reserve of granulocyte, due to suppression of bone marrow. Text Reference - p. 618

An elderly patient is diagnosed with pneumonia. The nurse reviews the patient's laboratory report, which reveals a normal WBC count. What are the likely reasons that the lab result does not correspond to the diagnosis of pneumonia? Select all that apply. 1 The laboratory reports are erroneous. 2 The patient has decreased T-cell function. 3 The patient has decreased humoral antibody response. 4 The blood sample might not have been obtained properly. 5 The patient has increased bone marrow reserve of granulocyte.

A patient with hypothyroidism is treat with levothyroxin (Synthroid). When teaching the patient about the therapy, the nurse a. explains that alternate-day dosage may be used if side effects occur b. provides written instructions for all information related to the medication therapy c. assures the patient that a return to normal function will occur with replacement therapy d. informs the patient that medications must be taken until hormone balance in reestablished

B- Because of the mental sluggishness, inattentiveness, and memory loss that occurs with hypothyroidism, it is important to provide written instructions and repeat information when teaching the patient. Replacement therapy must be taken for life, and alternate-day dosing is not therapeutic. Although most patients return to a normal state with treatment, cardiovascular conditions and psychoses may persist.

When caring for a patient with nephrogenic diabetes insipidus, the nurse would expect treatment to include a. fluid restriction b. thiazide diuretics c. a high sodium diet d. chlorprpamide (Diabinese)

B- In nephrogenic diabetes insipidus, the kidney is unable to respond to ADH, so vasopressin or hormone analogs are not effective. Thiazide diuretics slow the glomerular filtration rate in the kidney and produce a decrease in urine output. Low sodium diets (<3g/day) are also thought to decrease urine output. Fluids are not restricted because the patient could easily become dehydrated

A patient with SIADH is treated with water restriction and administration of IV fluids. The nurse evaluates that treatment has been effective when the patient experiences a. increased urine output, decreased serum sodium, and increased urine specific gravity b. increased urine output, increased serum sodium, and decreased urine specific gravity c. decreased urine output, increased serum sodium, and decreased urine specific gravity d. decreased urine output, decreased serum sodium, and increased urine specific gravity

B- The patient with SIADH has water retention with hyponatremia, decreased urine output, and concentrated urine with high specific gravity. Improvement in the patient's condition is reflected by increased urine output, normalization of serum sodium, and more water in the urine, decreasing specific gravity.

An appropriate nursing intervention for the patient with hyperparathyroidism is to a. pad side rails as a seizure precaution b. increase fluid intake to 3000 to 4000 mL daily c. maintain bed rest to prevent pathologic fractures d. monitor the patient for Trousseau's phenomenon and Chvostek's sign

B. A high fluid intake is indicated in hyperparathyroidism to dilute hypercalcemia and flush the kindeys so that calcium stone formation is reduced. Seizures are not associated with hyperparathyroidism, but impending tetany of hypoparathyroidism can be noted with Trousseau's phenomenon and Chvostek's sign. The patient with hyperparathyroidism is at risk for pathologic fractures resulting from decreased bone density, but mobility is encouraged to promote bone calcification.

The nurse is teaching a wellness class to a group of women at their workplace. The nurse knows that which woman is at highest risk for developing cancer? A. A woman who obtains regular cancer screenings and consumes a high-fiber diet B. A woman who has a body mass index of 35 kg/m2 and smoked cigarettes for 20 years C. A woman who exercises five times every week and does not consume alcoholic beverages D. A woman who limits fat consumption and has regular mammography and Pap screenings

B. A woman who has a body mass index of 35 kg/m2 and smoked cigarettes for 20 years Cancer prevention and early detection are associated with the following behaviors: limited alcohol use

and blue mucous membranes

B. Brittle nails

Which finding allows you to identify the patient's anemia as folic acid deficiency rather than cobalamin deficiency? A. Loss of appetite B. Lack of neuromuscular symptoms C. Red tongue D. Change in nail shape

B. Lack of neuromuscular symptoms The absence of neurologic problems is an important diagnostic finding and differentiates folic acid deficiency from cobalamin deficiency. Reference: 669

When the patient with parathyroid disease experiences symptoms of hypocalcemia, a measure that can be used to raise calcium levels temporarily is to a. administer IV normal saline b. have the patient rebreathe in a paper bag c. administer furosemid (Lasix) as ordered d. administer oral phosphorus supplements

B. Rebreathing in a paper bag promotes carbon dioxide retention in the blood, which lowers pH and creates acidosis. An acidemia enhances the solubility and ionization of calcium, increasing the proportion of total body calcium available in physiologically active form and relieving the symptoms of hypocalcemia. Saline promotes calcium excretion, as does furosemide. Phosphate levels in the blood are reciprocal to calcium, and an increase in phosphates promotes calcium excretion.

What can the nurse do to facilitate cancer prevention for the patient in the promotion stage of cancer development? A. Teach the patient to exercise daily. B. Teach the patient promoting factors to avoid. C. Tell the patient to have the cancer surgically removed now. D. Teach the patient which vitamins will improve the immune system.

B. Teach the patient promoting factors to avoid. The promotion stage of cancer is characterized by the reversible proliferation of the altered cells. Changing the lifestyle to avoid promoting factors (dietary fat, obesity, cigarette smoking, and alcohol consumption) can reduce the chance of cancer development. Daily exercise and vitamins alone will not prevent cancer. Surgery at this stage may not be possible without a critical mass of cells, and this advice would not be the nurse's role.

The patient is receiving an IV vesicant chemotherapy drug. The nurse notices swelling and redness at the site. What should the nurse do first? A. Ask the patient if the site hurts. B. Turn off the chemotherapy infusion. C. Call the ordering health care provider. D. Administer sterile saline to the reddened area.

B. Turn off the chemotherapy infusion. Because extravasation of vesicants may cause severe local tissue breakdown and necrosis, with any sign of extravasation the infusion should first be stopped. Then the protocol for the drug-specific extravasation procedures should be followed to minimize further tissue damage. The site of extravasation usually hurts, but it may not. It is more important to stop the infusion immediately. The health care provider may be notified by another nurse while the patient's nurse starts the drug-specific extravasation procedures, which may or may not include sterile saline.

The primary pathophysiology underlying thalassemia is A. erythropoietin deficiency. B. abnormal hemoglobin synthesis. C. autoimmunity. D. S-shaped hemoglobin.

B. abnormal hemoglobin synthesis. Thalassemia is a group of autosomal recessive diseases that involve inadequate production of normal hemoglobin. Hemolysis also occurs in thalassemia, but insufficient production of normal hemoglobin is the predominant problem. Erythropoietin deficiency is associated with a renal disorder, and S-shaped hemoglobin is associated with sickle cell disease. Reference: 668

Caring for a patient with a diagnosis of polycythemia vera will likely require you to A. encourage deep breathing and coughing. B. assist with or perform phlebotomy at the bedside. C. teach the patient how to maintain a low-activity lifestyle. D. perform thorough and regularly scheduled neurologic assessments.

B. assist with or perform phlebotomy at the bedside. Primary polycythemia often requires phlebotomy to reduce blood volume. The increased risk of thrombus formation that accompanies the disease requires regular exercises and ambulation. Deep-breathing and coughing exercises do not directly address the cause or common sequelae of polycythemia, and neurologic manifestations are not typical. Reference: 678

The nursing student asks the clinical instructor to explain why clinical symptoms are more important than laboratory values when the patient has experienced blood loss. The instructor correctly recognizes that A. nurses should focus on clinical symptoms because those are the parameters of nursing practice. B. blood values are often normal or even high because fluid shifts have not occurred and laboratory values are falsely high. C. laboratory values are used to supplement nursing assessments. D. laboratory findings are often falsely low in the early period of blood loss.

B. blood values are often normal or even high because fluid shifts have not occurred and laboratory values are falsely high. It is essential to understand that the clinical signs and symptoms the patient is experiencing are more important than the laboratory values. For example, an adult with a bleeding peptic ulcer who had a 750-mL hematemesis (15% of a normal total blood volume) within the past 30 minutes may have postural hypotension but have normal hemoglobin and hematocrit values. Over the ensuing 36 to 48 hours, most of the blood volume deficit will be replaced by the movement of fluid from the extravascular into the intravascular space. Only at these later times will the hemoglobin and hematocrit values reflect the blood loss. Reference: 671

When obtaining assessment data from a patient with a microcytic, hypochromic anemia, you question the patient about A. folic acid intake. B. dietary intake of iron. C. a history of gastric surgery. D. a history of sickle cell anemi

B. dietary intake of iron. Iron deficiency anemia is a type of microcytic, hypochromic anemia. Reference: 662

When caring for a patient with metastatic cancer, you note a hemoglobin level of 8.7 g/dL and hematocrit of 26%. You place highest priority on initiating interventions that can reduce A. thirst. B. fatigue. C. headache. D. abdominal pain.

B. fatigue. The patient with a low hemoglobin level and hematocrit is anemic and is most likely to experience fatigue. Fatigue develops because of the lowered oxygen-carrying capacity that leads to reduced tissue oxygenation with which to carry out cellular functions. Reference: 662

A patient with a diagnosis of hemophilia fell down an escalator earlier in the day and is now experiencing bleeding in her left knee joint. Your immediate response should include A. immediate transfusion of platelets. B. resting the patient's knee to prevent hemarthroses. C. assistance with intracapsular injection of corticosteroids. D. range-of-motion exercises to prevent thrombus formation.

B. resting the patient's knee to prevent hemarthroses. In patients with hemophilia, joint bleeding requires resting of the joint to prevent deformities from hemarthrosis. Clotting factors, not platelets or corticosteroids, are administered. Thrombus formation is not a central concern in a patient with hemophilia. Reference: 685-686

A patient is admitted to the hospital in thyrotoxic crisis. On physical assessment of the patient, the nurse would expect to find a. hoarseness and laryngeal stridor b. bulging eyeballs and dysrhythmias c. elevated temperature and signs of heart failure d. lethargy progressing suddenly to impairment of consciousness

C- A hyperthyroid crisis results in marked manifestations of hyperthyroidism, with severe tachycardia, heart failure, shock, hyperthermia, agitation, nausea, vomiting, diarrhea, delirium, and coma. Although exopthalmus may be present in the patient with Grave's disease, it is not a significant factor in hyperthyroid crisis. Hoarseness and laryngeal stridor are characteristic of tetany of hypoparathyroidism, and lethargy progressing to coma is characteristic of myxedema coma, a complication of hypothyroidism.

A patient with diabetes insipidus is treated with nasal demospressin (DDAVP). The nurse determines that the drug is not having an adequate therapeutic effect when the patient experiences a. headache and weight gain b nasal irritation and nausea c. urine specific gravity of 1.002 d. an oral intake greater than urinary output

C- Normal urine specific gravity of 1.003 to 1.030, and urine with a specific gravity of 1.002 is very dilute, indicating that there continues to be excessive water loss and that treatment of diabetes insipidus is inadequate. Headache, weight gain, and oral intake greater than urinary output are signs of volume excess that occur with over medication. Nasal irritation and nausea may also indicate overdosage

During assessment of the patient with acromegaly, the nurse would expect the patient to report a. infertility b. dry, irritated skin c. undesirable changes in appearance d. an increase in height of 2 to 3 inches a year

C- the increased production of GH in acromegaly causes an increase in thickness and width of bones and enlargement of soft tissues, resulted in marked changes in facial features, oily and coarse skin, and speech difficulties. Height is not increased in adults with GH excess because the epiphyses of the bones are closed

The patient has osteosarcoma of the right leg. The unlicensed assistive personnel (UAP) reports that the patient's vital signs are normal, but the patient says he still has pain in his leg and it is getting worse. What assessment question should the nurse ask the patient to determine treatment measures for this patient's pain? A. "Where is the pain?" B. "Is the pain getting worse?" C. "What does the pain feel like?" D. "Do you use medications to relieve the pain?"

C. "What does the pain feel like?" The unlicensed assistive personnel (UAP) told the nurse the location of the patient's pain and the worsening of pain (pattern). Asking about the quality of the pain will help in planning further treatment. The nurse should already know if the patient is using medication to relieve the pain or can check the patient's medication administration record to see if analgesics have been administered. The intensity of pain using a pain scale should also be assessed.

The female patient is having whole brain radiation for brain metastasis. She is concerned about how she will look when she loses her hair. What is the best response by the nurse to this patient? A. "When your hair grows back it will be patchy." B. "Don't use your curling iron and that will slow down the loss." C. "You can get a wig now to match your hair so you will not look different." D. "You should contact "Look Good, Feel Better" to figure out what to do about this."

C. "You can get a wig now to match your hair so you will not look different." Hair loss with radiation is usually permanent. The best response by the nurse is to suggest getting a wig before she loses her hair so she will not look or feel so different. When hair grows back after chemotherapy, it is frequently a different color or texture. Avoiding use of electric hair dryers, curlers, and curling irons may slow the hair loss but will not answer the patient's concern. The American Cancer Society's "Look Good, Feel Better" program will be helpful, but this response is avoiding the patient's immediate concern.

The patient was told that he would have intraperitoneal chemotherapy. He asks the nurse when the IV will be started for the chemotherapy. What should the nurse teach the patient about this type of chemotherapy delivery? A. It is delivered via an Ommaya reservoir and extension catheter. B. It is instilled in the bladder via a urinary catheter and retained for 1 to 3 hours. C. A Silastic catheter will be percutaneously placed into the peritoneal cavity for chemotherapy administration. D. The arteries supplying the tumor are accessed with surgical placement of a catheter connected to an infusion pump.

C. A Silastic catheter will be percutaneously placed into the peritoneal cavity for chemotherapy administration. Intraperitoneal chemotherapy is delivered to the peritoneal cavity via a temporary percutaneously inserted Silastic catheter and drained from this catheter after the dwell time in the peritoneum. The Ommaya reservoir is used for intraventricular chemotherapy. Intravesical bladder chemotherapy is delivered via a urinary catheter. Intraarterial chemotherapy is delivered via a surgically placed catheter that delivers chemotherapy via an external or internal infusion pump.

During discharge teaching for the patient with Addison's disease, the nurse identifies the need for additional instruction when the patient says, a. "I should always call the doctor is I develop vomiting or diarrhea" b. "If my weight goes down, my dosage of steroid is probably too high" c. "I should double or triple my steroid dose if I undergo rigorous physical exercise" d. "I need to carry an emergency kit with injectable hydrocortisone in case I can't take my medication by mouth"

C. A weight reduction in the patient with Addison's disease may indicate a fluid loss and a dose of replacement therapy that is too low rather than too high. Patient's with Addison's disease are taught to take two to three times their usual dose of steroids if they become ill, have teeth extracted, or engage in rigorous physical activity and should always have injectable hydrocortisone available if oral doses cannot be taken. Because vomiting and diarrhea are early signs of crisis and because fluid and electrolytes must be replaced, patients should notify their health care provider if these symptoms occur.

Which nursing intervention should you prioritize in the care of a 30-year-old woman who has a diagnosis of immune thrombocytopenic purpura (ITP)? A. Administration of packed red blood cells B. Administration of clotting factors VIII and IX C. Administration of oral or intravenous corticosteroids D. Maintenance of reverse isolation and application of standard precautions

C. Administration of oral or intravenous corticosteroids Common treatment modalities for ITP include corticosteroid therapy to suppress the phagocytic response of splenic macrophages. Blood transfusions, administration of clotting factors, and reverse isolation are not interventions that are indicated in the care of patients with ITP. Reference: 680

A patient is scheduled for a bilateral adrenalectomy. During the postoperative period, the nurse would expect administration of corticosteroids to be a. reduced to promote wound healing b. withheld until symptoms of hypocortisolism appear c. increased to promote an adequate response to the stress of surgery d. reduced because excessive hormones are released during surgical manipulation of the glands

C. Although the patient with Cushing syndrome has excess corticosteroids, removal of the glands the stress of surgery require that high doses of cortisone be administered postoperatively for several days before weaning the dose. The nurse should monitor the patient postoperatively to detect whether large amounts of hormones were released during surgical manipulation and to ensure that healing is satisfactory.

The patient with breast cancer is having teletherapy radiation treatments after her surgery. What should the nurse teach the patient about the care of her skin? A. Use Dial soap to feel clean and fresh. B. Scented lotion can be used on the area. C. Avoid heat and cold to the treatment area. D. Wear the new bra to comfort and support the area.

C. Avoid heat and cold to the treatment area. Avoiding heat and cold in the treatment area will protect it. Only mild soap and unscented, nonmedicated lotions may be used to prevent skin damage. The patient will want to avoid wearing tight-fitting clothing such as a bra over the treatment field and will want to expose the area to air as often as possible.

Which cellular dysfunction in the process of cancer development allows defective cell proliferation? A. Proto-oncogenes B. Cell differentiation C. Dynamic equilibrium D. Activation of oncogenes

C. Dynamic equilibrium Dynamic equilibrium is the regulation of proliferation that usually only occurs to equal cell degeneration or death or when the body has a physiologic need for more cells. Cell differentiation is the orderly process that progresses a cell from a state of immaturity to a state of differentiated maturity. Mutations that alter the expression of proto-oncogenes can activate them to function as oncogenes, which are tumor-inducing genes and alter their differentiation.

In addition to altered red blood cells (RBCs), which laboratory finding does the nurse expect for the patient with sickle cell disease? A. Leukocytosis B. Hypouricemia C. Hyperbilirubinemia D. Hypercholesteremia

C. Hyperbilirubinemia As a result of accelerated RBC breakdown, the patient may have characteristic clinical findings of hemolysis, including jaundice and elevated serum bilirubin levels. Reference: 674

The nurse is caring for a patient receiving an initial dose of chemotherapy to treat a rapidly growing metastatic colon cancer. The nurse is aware that this patient is at risk for tumor lysis syndrome (TLS) and will monitor the patient closely for which abnormality associated with this oncologic emergency? A. Hypokalemia B. Hypouricemia C. Hypocalcemia D. Hypophosphatemia

C. Hypocalcemia TLS is a metabolic complication characterized by rapid release of intracellular components in response to chemotherapy. This can rapidly lead to acute renal injury. The hallmark signs of TLS are hyperuricemia, hyperphosphatemia, hyperkalemia, and hypocalcemia.

You expect which laboratory finding to be abnormal for a patient with hemochromatosis? A. RBCs B. Platelets C. Iron D. Folic acid

C. Iron The normal range for total body iron is 2 to 6 g. Individuals with hemochromatosis accumulate iron at a rate of 0.5 to 1.0 g each year and may exceed total iron concentrations of 50 g. The other values are near normal. Reference: 676

A 33-year-old patient has recently been diagnosed with stage II cervical cancer. What should the nurse understand about the patient's cancer? A. It is in situ. B. It has metastasized. C. It has spread locally. D. It has spread extensively.

C. It has spread locally. Stage II cancer is associated with limited local spread. Stage 0 denotes cancer in situ

The patient is told that the adenoma tumor is not encapsulated but has normally differentiated cells and that surgery will be needed. The patient asks the nurse what this means. What should the nurse tell the patient? A. It will recur. B. It has metastasized. C. It is probably benign. D. It is probably malignant.

C. It is probably benign. Benign tumors usually are encapsulated and have normally differentiated cells. They do not metastasize and rarely recur as malignant tumors do.

Which organ is at greatest risk due to the effects of hemolytic anemia? A. Heart B. Spleen C. Kidney D. Liver

C. Kidney For all causes of hemolysis, a major focus of treatment is to maintain renal function. When RBCs are hemolyzed, the hemoglobin molecule is released and filtered by the kidneys. The accumulation of hemoglobin molecules can obstruct the renal tubules and lead to acute tubular necrosis Reference: 672

When providing teaching for the patient with iron-deficiency anemia who has been prescribed iron supplements, you should include taking the iron with which beverage? A. Milk B. Ginger ale C. Orange juice D. Water

C. Orange juice Taking iron with vitamin C (ascorbic acid) or orange juice, which contains ascorbic acid, also enhances iron absorption. Milk may interfere with iron absorption. Ginger ale and water do not facilitate iron absorption. Reference: 667

A 64-year-old male patient who is receiving radiation to the head and neck as treatment for an invasive malignant tumor complains of mouth sores and pain. Which intervention should the nurse add to this patient's plan of care? A. Weigh the patient every month to monitor for weight loss. B. Cleanse the mouth every 2 to 4 hours with hydrogen peroxide. C. Provide high-protein and high-calorie, soft foods every 2 hours. D. Apply palifermin (Kepivance) liberally to the affected oral mucosa.

C. Provide high-protein and high-calorie, soft foods every 2 hours. A patient with stomatitis should have soft, nonirritating foods offered frequently. The diet should be high in protein and high in calories. Saline or water should be used to cleanse the mouth (not hydrogen peroxide). Palifermin is administered intravenously as a growth factor to stimulate cells on the surface layer of the mouth to grow. Patients should be weighed at least twice each week to monitor for weight loss.

Which findings do you expect to find for a patient with acute loss of blood? A. Weakness, lethargy, and warm, dry skin B. Restlessness, hyperthermia, and bradycardia C. Tachycardia, hypotension, and cool, clammy skin D. Widened pulse pressure, anxiety, and hypoventilation

C. Tachycardia, hypotension, and cool, clammy skin Tachycardia, hypotension, and cool, clammy skin can be found in a person who has had an acute loss of blood. These are manifestations of hypovolemic shock. A person with a bleeding peptic ulcer who had a 750-mL hematemesis (15% of a normal total blood volume) within the past 30 minutes may have postural hypotension. Because blood is shunted to major organs, the skin in the periphery is cool to the touch. Tachycardia is the body's attempt to maintain adequate cardiac output. Reference: 671

A patient with hypoparathyroidism resulting form surgical treatment of hyperparathyroidism is preparing for discharge. The nurse teaches the patient that a. milk and milk products should be increased in the diet b. parenteral replacement of parathyroid hormone (PTH) will be required for life c. calcium supplements with vitamin D can effectively maintain calcium balance d. bran and whole-grain foods should be used to prevent GI effects of replacement therapy

C. The hypocalcemia that results from parathyroid hormone (PTH) deficiency is controlled with calcium and vitamin D supplementation and possibly oral phosphate binders. Replacement with PTH is not used because of antibody formation to PTH, the need for parenteral administration, and cost. Milk products, although good sources of calcium, also have high levels of phosphate, which reduce calcium absorption. Whole grains and foods containing oxalic acid also impair calcium absorption.

A patient with Addison's disease comes to the emergency department with complaints of nausea, vomiting, diarrhea, and fever. The nurse would expect collaborative care to include a. parenteral injections of adrenocorticotropic hormone (ACTH) b. IV administrations of vasopressors c. IV administration of hydrocortisone d. IV administration of D5W with 20 mEq KCL

C. Vomiting and diarrhea are early indicators of addisonian crisis, and fever indicates infection, which is causing additional stress for the patient. Treatment of a crisis requires immediate glucocorticoid replacement, and IV hydrocortisone, fluids, sodium, and glucose are necessary for 24 hours. Addison's disease is a primary insufficiency of the adrenal gland, and adrenocorticotropic hormone (ACTH) is not effective, nor would vasopressors be effective with fluid deficiency of Addison's disease. Potassium levels are increased in Addison's disease, and KCl would be contraindicated.

You anticipate the patient with hemochromatosis to be from which ethnic group? A. African American B. Hispanic American C. White European D. Chinese

C. White European Hemochromatosis is the most common genetic disorder among whites, with an incidence of 3 to 5 cases per 1000 whites of European ancestry. Reference: 676

You anticipate the onset of manifestations related to thalessemia to occur by A. 6 months of age. B. age 1 year. C. age 2 year. D. adolescence.

C. age 2 year. The manifestations, including growth and developmental deficits, develop in childhood by 2 years of age. Reference: 668

A patient with Graves' disease asks the nurse what caused the disorder. What is the best response by the nurse? a. "The cause of Graves' disease is not known, although it is thought to be genetic." b. "It is usually associated with goiter formation from an iodine deficiency over a long period of time." c. "Antibodies develop against thyroid tissue and destroy it, causing a deficiency of thyroid hormones." d. "In genetically susceptible persons, antibodies are formed that cause excessive thyroid hormone secretion."

d. In Graves' disease, antibodies to the TSH receptor are formed, attach to the receptors, and stimulate the thyroid gland to release triiodothyronine (T3), thyroxine (T4), or both, creating hyperthyroidism. The disease is not directly genetic but individuals appear to have a genetic susceptibility to develop autoimmune antibodies. Goiter formation from insufficient iodine intake is usually associated with hypothyroidism.

What characteristic is related to Hashimoto's thyroiditis? a. Enlarged thyroid gland b. Viral-induced hyperthyroidism c. Bacterial or fungal infection of thyroid gland d. Chronic autoimmune thyroiditis with antibody destruction of thyroid tissue

d. In Hashimoto's thyroiditis, thyroid tissue is destroyed by autoimmune antibodies. An enlarged thyroid gland is a goiter. Viral-induced hyperthyroidism is subacute granulomatous thyroiditis. Acute thyroiditis is caused by bacterial or fungal infection.

A patient who recently had a calcium oxalate renal stone had a bone density study, which showed a decrease in her bone density. What endocrine problem could this patient have? a. SIADH c. Cushing syndrome b. Hypothyroidism d. Hyperparathyroidism

d. The patient with hyperparathyroidism may have calcium nephrolithiasis, skeletal pain, decreased bone density, psychomotor retardation, or cardiac dysrhythmias. The other endocrine problems would not be related to calcium kidney stones or decreased bone density.

When providing discharge instructions to a patient who had a subtotal thyroidectomy for hyperthyroidism, what should the nurse teach the patient? a. Never miss a daily dose of thyroid replacement therapy. b. Avoid regular exercise until thyroid function is normalized. c. Use warm saltwater gargles several times a day to relieve throat pain. d. Substantially reduce caloric intake compared to what was eaten before surgery.

d. With the decrease in thyroid hormone postoperatively, calories need to be reduced substantially to prevent weight gain. When a patient has had a subtotal thyroidectomy, thyroid replacement therapy is not given because exogenous hormone inhibits pituitary production of TSH and delays or prevents the restoration of thyroid tissue regeneration. Regular exercise stimulates the thyroid gland and is encouraged. Saltwater gargles are used for dryness and irritation of the mouth and throat following radioactive iodine therapy.

Prediabetes

defined as impaired glucose tolerance (IGT), impaired fasting glucose (IFG), or both. It is an intermediate stage between normal glucose homeostasis and diabetes, in which the blood glucose levels are elevated but not high enough to meet the diagnostic criteria for diabetes.

Gestational diabetes

develops during pregnancy and occurs in about 4.6% to 9.2% of pregnancies in the United States.8 Women with gestational diabetes have a higher risk for cesarean delivery, and their babies have increased risk for perinatal death, birth injury, and neonatal complications.

Insulin

hormone produced by the β cells in the pancreas - promotes glucose transport from the bloodstream across the cell membrane to the cytoplasm of the cell - lowers blood glucose level

Clinical manifestations of hyperthyroidism occur as a result of..

increased metabolic rate and tissue sensitivity to the sympathetic nervous system

Hyperosmolar hyperglycemic syndrome (HHS)

is a life-threatening syndrome that can occur in the patient with diabetes who is able to produce enough insulin to prevent DKA, but not enough to prevent severe hyperglycemia, osmotic diuresis, and extracellular fluid depletion

Diabetic Ketoacidosis

is caused by a profound deficiency of insulin and is characterized by hyperglycemia, ketosis, acidosis, and dehydration.

Type 2 Diabetes - Etiology and pathophysiology

is characterized by a combination of inadequate insulin secretion and insulin resistance. The pancreas usually produces some endogenous (self-made) insulin. However, the body either does not produce enough insulin or does not use it effectively, or both.

insulin-dependent tissues

muscle and adipose tissue - insulin is required to "unlock" these receptor sites, allowing the transport of glucose into the cells to be used for energy

Moderate Activity (200-350 kcal/hr)

• Active housework • Bicycling (light) • Bowling • Dancing • Gardening • Golf • Roller skating • Walking briskly

Vigorous Activity (400-900 kcal/hr

• Aerobic exercise • Bicycling (vigorous) • Hard labor • Ice skating • Outdoor sports • Running • Soccer • Tennis • Wood chopping

Light Activity (100-200 kcal/hr)

• Fishing • Light housework • Secretarial work • Teaching • Walking casually

Prevention and Early Detection of Diabetes Mellitus

• Increase level of exercise because physical activity reduces the risk of type 2 diabetes. • Maintain a healthy weight because obesity increases the risk of type 2 diabetes. • If overweight, lose weight and participate in a regular exercise program to reduce the risk of diabetes. • Choose foods that are low in fat content, total calories, and processed foods and high in whole grains, fruits, and vegetables. • If overweight and over age 45, get screened for diabetes.

Nursing Diagnoses

• Ineffective health management related to deficient knowledge of diabetes management and lack of understanding of diabetes management plan • Risk for unstable blood glucose levels related to infrequent blood glucose monitoring and lack of following diabetes management plan • Risk for injury related to decreased tactile sensation, episodes of hypoglycemia • Risk for peripheral neurovascular dysfunction related to vascular effects of diabetes

Cultural & Ethnic Health Disparities - Diabetes Mellitus

• The highest incidence of diabetes is among Native Americans and Alaska Natives (16.5% are being treated for diabetes). • Pima Indians in Arizona have the highest rate of diabetes in the world (50% of adults have diabetes). • The rates of diabetes are: non-Hispanic whites 7.1%, Asian Americans 8.4%, Hispanics 11.8%, and non-Hispanic blacks 12.6%. • Diabetes is 1.5 times more likely to cause death in Hispanics and 2.2 times more likely to cause death in African Americans than in non-Hispanic whites. • Complications from diabetes are a major cause of death among Native Americans. Native Americans have a six times higher rate of end-stage renal disease and a four times higher rate of amputation than other ethnicities with diabetes.

Evaluation

• Verbalize key elements of the therapeutic regimen, including knowledge of disease and treatment plan • Describe self-care measures that may prevent or slow progression of chronic complications • Maintain a balance of nutrition, activity, and insulin availability that results in stable, safe, and healthy blood glucose levels • Experience no injury resulting from decreased sensation in feet • Implement measures to increase peripheral circulation


Kaugnay na mga set ng pag-aaral

What is unique about The DNA of a prokaryote

View Set

Studies Weekly 18 - The Germanic Tribes

View Set

OTA BIO WEEK 7 CHAPTER 9 muscles exercise

View Set